» »

Antibiotice. Utilizarea antibioticelor

27.04.2019

Astăzi chiar și copiii din școala primară știu ce sunt antibioticele. Cu toate acestea, termenul „antibiotice” gamă largă acțiuni” uneori derutează chiar și adulții și ridică multe întrebări. Cât de larg este spectrul? Ce antibiotice sunt acestea? Și, da, se pare că există și medicamente cu spectru îngust care s-ar putea să nu ajute?

Cel mai surprinzător lucru este că, deseori, chiar și Internetul atotștiutor nu poate ajuta și risipi ceața îndoielii. În acest articol vom încerca să ne dăm seama încet și metodic ce fel de antibiotice cu spectru larg sunt acestea, asupra ce bacterii acționează, precum și când, cum și de câte ori pe zi sunt utilizate.

Lumea diversă a bacteriilor

Și vom începe de la bun început - cu microbi. Bacteriile alcătuiesc majoritatea procariotelor - organisme vii unicelulare fără un nucleu clar definit. Bacteriile au fost cele care au populat pentru prima dată pământul singuratic cu milioane de ani în urmă. Ei trăiesc peste tot: în sol, apă, izvoare termale acide și deșeuri radioactive. Sunt cunoscute descrieri a aproximativ 10 mii de specii de bacterii, dar se estimează că numărul acestora ajunge la un milion.

Și, desigur, bacteriile trăiesc în corpurile plantelor, animalelor și oamenilor. Relațiile dintre organismele unicelulare inferioare și organismele multicelulare superioare pot fi diferite - ambele prietenoase, reciproc avantajoase pentru parteneri și deschis ostile.

O persoană nu poate exista fără bacterii „bune”, corecte, care formează microflora. Cu toate acestea, împreună cu bifidobacterii și lactobacili valoroase, microbii care provoacă o mare varietate de boli intră în corpul nostru.

Microflora include și așa-numitele microorganisme oportuniste. În condiții favorabile, nu fac rău, dar de îndată ce imunitatea noastră scade, acești prieteni de ieri se transformă în dușmani vicioși. Pentru a înțelege cumva gazda bacteriilor, medicii au propus să le clasifice.

Gram- și Gram+: descifrarea puzzle-ului

Cea mai faimoasă diviziune a microbilor este foarte des menționată în farmacii, clinici și în adnotările de medicamente. Și la fel de des, pacientul mediu în viață nu înțelege despre ce vorbim de fapt. Să ne dăm seama împreună ce înseamnă aceste expresii misterioase gram+ și gram-, fără de care nici o singură descriere a acțiunii antibioticelor nu este completă?

În 1885, danezul Hans Gram a decis să coloreze secțiunile țesut pulmonar pentru a face bacteriile mai vizibile. Omul de știință a descoperit că agentul cauzator al tifosului, Salmonella typhi, nu și-a schimbat culoarea, în timp ce alte microorganisme au fost expuse la substanța chimică.

Cea mai faimoasă clasificare de astăzi se bazează pe capacitatea bacteriilor de a colora conform Gram. Un grup de bacterii care nu își schimbă culoarea se numește gram-negative. A doua categorie se numește gram-pozitive, adică microorganisme cu colorație Gram.

Agenții patogeni gram-pozitivi și gram-negativi: cine este cine?

O altă clasificare, nu mai puțin importantă, a antibioticelor descompune medicamentele în funcție de spectrul lor de acțiune și structura. Din nou, pentru a înțelege paragrafele complexe ale instrucțiunilor care explică spectrul de activitate și aparțin unui anumit grup, trebuie să cunoașteți mai bine microbii.

Bacteriile Gram-pozitive includ coci, adică microorganisme sferice, inclusiv numeroase familii de stafilococi și streptococi. În plus, clostridiilor, corynebacteria, listeria și enterococii aparțin acestui grup. Agenții patogeni Gram pozitivi cauzează cel mai adesea boli infecțioase ale nazofaringelui, tractului respirator, ureche, precum și procesele inflamatorii ale ochiului.

Bacteriile Gram-negative sunt un grup nu atât de numeros de microorganisme care provoacă în principal infecții intestinale, precum și boli ale tractului genito-urinar. Mult mai rar, agenții patogeni gram-negativi sunt responsabili pentru patologiile tractului respirator. Acestea includ Escherichia coli, Salmonella, Shigella (agentul cauzal al difteriei), Pseudomonas, Moraxella, Legionella, Klebsiella, Proteus.

Printre microorganismele gram-negative există și agenți cauzali ai infecțiilor severe ale spitalelor. Acești microbi sunt greu de tratat - în condițiile spitalicești dezvoltă rezistență specială la majoritatea antibioticelor. Prin urmare, antibiotice speciale, adesea intramusculare sau intravenoase, cu spectru larg sunt utilizate pentru a trata astfel de boli infecțioase.

Terapia empirică se bazează pe această „separare” a bacteriilor gram-negative și gram-pozitive, care implică selectarea unui antibiotic fără cultură prealabilă, adică practic „cu ochi”. După cum arată practica, în cazul bolilor „standard”, această abordare a alegerii unui medicament este complet justificată. Dacă medicul are îndoieli cu privire la faptul că agentul patogen aparține unui grup sau altuia, prescrierea de antibiotice cu spectru larg va ajuta „să puneți mingea în aer”.

Antibiotice cu spectru larg: toată armata este sub amenințarea armei

Așadar, ajungem la partea cea mai interesantă. Antibioticele cu spectru larg sunt un medicament antibacterian universal. Oricare ar fi agentul patogen care este sursa bolii, agenții antibacterieni cu spectru larg vor avea un efect bactericid și vor învinge microbul.

De regulă, medicamentele cu spectru larg sunt utilizate atunci când:

  • tratamentul este prescris empiric, adică pe baza simptomelor clinice. Atunci când se selectează un antibiotic în mod empiric, timpul și banii nu sunt irosite în identificarea agentului patogen. Microbul care a provocat boala va rămâne pentru totdeauna necunoscut. Această abordare este adecvată în cazul infecțiilor comune, precum și al bolilor periculoase cu acțiune rapidă. De exemplu, cu meningită moarte poate fi o concluzie prealabilă literalmente în câteva ore dacă terapia cu antibiotice nu este începută imediat după primele semne ale bolii;
  • agenții patogeni sunt rezistenți la antibioticele cu spectru îngust;
  • a fost diagnosticată o suprainfecție, în care mai multe tipuri de bacterii sunt vinovatele bolii;
  • prevenirea infecției după intervenții chirurgicale.

Lista antibioticelor cu spectru larg

Să încercăm să numim pe nume acele medicamente antibacteriene care au un spectru larg de activitate:

  • antibiotice din grupa penicilinei: , Ampicilină, Ticarciclină;
  • antibiotice din grupa tetraciclinelor: tetracicline;
  • fluorochinolone: ​​Levofloxacin, Gatifloxacin, Moxifloxacin, Ciprofloxacin;
  • Aminoglicozide: Streptomicina;
  • Amfenicoli: Cloramfenicol (Levomicetina);
  • Carbapeneme: Imipenem, Meropenem, Ertapenem.

După cum puteți vedea, lista antibioticelor cu spectru larg nu este foarte mare. Și vom începe o descriere detaliată a medicamentelor cu probabil cel mai popular grup - antibioticele peniciline.

Penicilinele - medicamente pe care oamenii le cunosc și le iubesc

Odată cu descoperirea unui antibiotic din acest grup special - benzilpenicilina - medicii și-au dat seama că microbii ar putea fi înfrânți. În ciuda vârstei sale venerabile, benzilpenicilina este folosită și astăzi, iar în unele cazuri este un medicament de primă linie. Cu toate acestea, agenții cu spectru larg includ alte antibiotice peniciline mai noi, care pot fi împărțite în două grupuri:

  • medicamente pentru administrare parenterală (injectabilă) și enterală, care rezistă mediului acid al stomacului;
  • antibiotice injectabile care nu rezistă la acțiunea acidului clorhidric - Carbenicilină, Ticarcilină.

Ampicilina și Amoxicilina sunt peniciline populare cu spectru larg

Ampicilina și Amoxicilina ocupă un loc de onoare deosebit în rândul antibioticelor peniciline. Spectrul și efectul asupra corpului uman al acestor două antibiotice sunt aproape aceleași. Dintre microorganismele sensibile la Ampicilină și Amoxicilină, cei mai cunoscuți agenți infecțioși sunt:

  • bacterii gram-pozitive: stafilococi și streptococi, enterococi, listeria;
  • bacterii gram-negative: patogen pentru gonoree Neisseria gonorrhoeae, E. coli, Shigella, salmonella, Haemophilus influenzae, patogen pentru tuse convulsivă Bordetella pertussis.

Cu un spectru identic, ampicilina și amoxicilina diferă semnificativ în proprietăți farmacocinetice.

Ampicilină

Ampicilina a fost sintetizată la începutul anilor 60 ai secolului trecut. Medicamentul a câștigat imediat inimile medicilor: spectrul său de acțiune se compara favorabil cu antibioticele din anii 50, față de care persistența, adică dependența, se dezvoltase deja.

Cu toate acestea, ampicilina are dezavantaje semnificative - biodisponibilitate scăzută și timp de înjumătățire scurt. Antibioticul este absorbit doar cu 35-50%, iar timpul de înjumătățire este de câteva ore. În acest sens, cursul tratamentului cu Ampicilină este destul de intens: comprimatele trebuie luate în doză de 250-500 mg de patru ori pe zi.

O caracteristică a Ampicilinei, care este considerată un avantaj față de Amoxicilină, este posibilitatea administrării parenterale a medicamentului. Antibioticul se produce sub forma unei pulberi liofilizate, din care se prepara o solutie inainte de administrare. Ampicilina se prescrie 250-1000 mg la fiecare 4-6 ore intramuscular sau intravenos.

Amoxicilina este puțin mai tânără decât predecesorul său - a fost pusă în vânzare în anii 70 ai secolului XX. Cu toate acestea, acest antibiotic este încă unul dintre cele mai populare și mijloace eficiente gamă largă, inclusiv pentru copii. Și acest lucru a devenit posibil datorită avantajelor incontestabile ale medicamentului.

Acestea includ biodisponibilitatea ridicată a comprimatelor de amoxicilină, care ajunge la 75–90%, pe fondul unui timp de înjumătățire destul de lung. Mai mult, gradul de absorbție nu depinde de aportul alimentar. Medicamentul are un grad ridicat de afinitate pentru țesuturile tractului respirator: concentrația de amoxicilină în plămâni și bronhii este aproape de două ori mai mare decât în ​​alte țesuturi și sânge. Nu este surprinzător faptul că amoxicilina este considerată medicamentul de elecție pentru formele necomplicate de bronșită bacteriană și pneumonie.

În plus, medicamentul este indicat pentru dureri în gât, infecții ale tractului urinar și reproducător și boli infecțioase ale pielii. Amoxicilina este o componentă a terapiei de eradicare pentru ulcer peptic stomac și duoden.

Medicamentul se administrează pe cale orală la o doză de 250-1000 mg de două ori pe zi timp de 5-10 zile.

Peniciline parenterale cu spectru larg

Penicilinele, care sunt utilizate pentru administrare parenterală, diferă de cunoscutele Ampicilină și Amoxicilină prin activitatea lor suplimentară împotriva Pseudomonas aeruginosa. Acest microorganism provoacă infecții ale țesuturilor moi - abcese, răni purulente. Pseudomonas acționează și ca agenți cauzali ai cistitei - inflamația vezicii urinare, precum și inflamația intestinelor - enterita.

În plus, antibioticele peniciline parenterale cu spectru larg au efecte bactericide și bacteriostatice împotriva:

  • microorganisme gram-pozitive: stafilococi, streptococi (cu excepția tulpinilor care formează penicilinaza), precum și enterobacterii;
  • microorganisme gram-negative: Proteus, Salmonella, Shigella, Escherichia coli, Haemophilus influenzae și altele.

Penicilinele parenterale cu spectru larg includ carbenicilina, ticarcilina, carfecilina, piperacilina și altele.

Să ne uităm la cele mai cunoscute antibiotice - Carbenicilină, Ticarcilină și Piperacilină.

Carbenicilina

În medicină se folosește sarea carbenicilinei disodice, care este o pulbere albă care se dizolvă înainte de utilizare.

Carbenicilina este indicată pentru infecții abdominale, inclusiv peritonită, sistemul genito-urinar, tractului respirator, precum și meningită, sepsis, infecții ale țesutului osos și ale pielii.

Medicamentul se administrează intramuscular și, în cazuri severe, intravenos.

Ticarcilină

Ticarcilina neprotejată este prescrisă pentru infecții severe cauzate de tulpini de bacterii care nu produc penicilinază: sepsis, septicemie, peritonită, infecții postoperatorii. Antibioticul este, de asemenea, utilizat pentru infecții ginecologice, inclusiv endometrită, precum și infecții ale tractului respirator, organelor ORL și ale pielii. În plus, Ticarcilina este utilizată pentru boli infecțioase la pacienții cu un răspuns imunitar redus.

Piperacilină

Piperacilina este utilizată în principal împreună cu inhibitorul beta-lactamazei tazobactam. Cu toate acestea, dacă se stabilește că agentul cauzal al bolii nu produce penicilinază, se poate prescrie un antibiotic neprotejat.

Indicațiile pentru utilizarea piperacilinei includ infecții purulente-inflamatorii severe ale sistemului genito-urinar, cavității abdominale, organelor respiratorii și ORL, piele, oase și articulații, precum și sepsis, meningită, infecții postoperatorii și alte boli.

Peniciline protejate cu spectru larg: antibiotice pentru combaterea rezistenței!

Amoxicilina și Ampicilina sunt departe de a fi omnipotente. Ambele medicamente sunt distruse de beta-lactamaze, care sunt produse de unele tulpini de bacterii. Astfel de agenți patogeni „dăunători” includ multe tipuri de stafilococ, inclusiv Staphylococcus aureus, Haemophilus influenzae, Moraxella, Escherichia coli, Klebsiella și alte bacterii.

Dacă infecția este cauzată de agenți patogeni producatori de beta-lactamaze, amoxicilină, ampicilină și alte antibiotice sunt pur și simplu distruse fără a dăuna bacteriilor. Oamenii de știință au găsit o cale de ieșire din situație prin crearea unor complexe de antibiotice peniciline cu substanțe care inhibă beta-lactamazele. Pe lângă cel mai faimos acid clavulanic, inhibitorii enzimelor distructive includ sulbactam și tazobactam.

Antibioticele protejate pot lupta eficient împotriva infecțiilor pe care penicilina fragilă și singuratică nu poate. Prin urmare, medicamentele combinate sunt adesea medicamentele de alegere pentru o mare varietate de boli cauzate de infecția bacteriană, inclusiv cele dobândite în spital. Locurile fruntașe în această listă de antibiotice cu spectru larg sunt ocupate de două sau trei medicamente, iar unele medicamente injectabile utilizate în spitale rămân în culise. Aducând un omagiu spectrului fiecărei peniciline combinate, vom deschide vălul secretului și vom enumera aceste medicamente, desigur, demne.

Amoxicilină + acid clavulanic. Cel mai faimos antibiotic combinat cu spectru larg, care are zeci de generice: Augmentin, Amoxiclav, Flemoclav. Există atât forme orale, cât și forme injectabile ale acestui antibiotic.


Amoxicilină și sulbactam. Denumirea comercială - Trifamox, disponibil sub formă de tablete. Este disponibilă și o formă parenterală de Trifamox.

Ampicilină și sulbactam. Denumirea comercială - Ampisid, folosit pentru injecții, mai des în spitale.

Ticarcilină + acid clavulanic. Denumirea comercială Timentin, disponibilă numai sub formă parenterală. Indicat pentru tratamentul infecțiilor severe cauzate de tulpini rezistente, dobândite în spital.

Piperacilină + tazobactam. Denumiri comerciale Piperacillin-tazobactam-Teva, Tazacin, Santaz, Tazrobida, Tacillin J etc. Antibioticul este utilizat ca picurare de perfuzie, adică sub formă de perfuzii intravenoase pentru poliinfecții moderate și severe.

Tetracicline cu spectru larg: testate în timp

La număr medicamente cunoscute Antibioticele tetracicline au, de asemenea, un spectru larg de acțiune. Acest grup de medicamente este unit de o structură comună, care se bazează pe sistemul patru ciclic („tetra” tradus din greacă - patru).

Antibioticele tetracicline nu au un inel beta-lactamic în structura lor și, prin urmare, nu sunt supuse acțiunii distructive a beta-lactamazei. Grupul de tetracicline are un spectru general de acțiune, care include:

  • microorganisme gram-pozitive: stafilococi, streptococi, clostridii, listeria, actinomicete;
  • microorganisme gram-negative: agentul cauzator al gonoreei Neisseria gonorrhoeae, Haemophilus influenzae, Klebsiella, Escherichia coli, Shigella (agentul cauzator al dizenteriei), salmonella, agentul cauzator al tusei convulsive Bordetella pertussis, precum și bacteriile genus Treponema, inclusiv agentul cauzal al sifilisului - spirochete pallidum.

O caracteristică distinctivă a tetraciclinelor este capacitatea lor de a pătrunde în celula bacteriană. Prin urmare, aceste produse fac față bine agenților patogeni intracelulari - chlamydia, micoplasma, ureaplasma. Pseudomonas aeruginosa și Proteus nu sunt susceptibile la acțiunea bactericidă a tetraciclinelor.

Cele două tetracicline cele mai utilizate astăzi sunt tetraciclina și doxiciclina.

Tetraciclină

Unul dintre fondatorii grupului de tetracicline, descoperit în 1952, este folosit și astăzi, în ciuda vârstei înaintate și a efectelor secundare. Totuși, prescrierea comprimatelor de tetraciclină poate fi criticată, având în vedere existența unor antibiotice cu spectru larg mai moderne și mai eficiente.

Aspectele negative ale tetraciclinei orale includ, fără îndoială, activitatea sa terapeutică destul de limitată, precum și capacitatea de a modifica compoziția florei intestinale. În acest sens, atunci când prescrieți comprimate de tetraciclină, trebuie să țineți cont risc crescut apariția diareei asociate antibioticelor.

Este mult mai eficient și mai sigur să prescrii forme externe și locale de tetraciclină. Astfel, unguentul pentru ochi cu tetraciclină este inclus în Lista rusă medicamente vitale și este un exemplu excelent de medicament antibacterian topic cu spectru larg.

Doxiciclina

Doxiciclina se remarcă prin activitatea sa terapeutică (de aproape 10 ori mai mare decât tetraciclina) și prin biodisponibilitatea impresionantă. În plus, doxiciclina are un efect mult mai mic asupra microflorei intestinale decât alte medicamente din grupul tetraciclinelor.

Fluorochinolonele sunt antibiotice esențiale cu spectru larg

Probabil că nici un medic nu-l va putea prezenta practică medicală fără antibiotice fluorochinolone. Primii reprezentanți sintetizați ai acestui grup s-au distins printr-un spectru restrâns de acțiune. Odată cu dezvoltarea produselor farmaceutice, au fost descoperite noi generații de fluorochinolone agenți antibacterieni iar gama activităților lor sa extins.

Astfel, antibioticele de prima generatie - Norfloxacin, Ofloxacin, Ciprofloxacin - actioneaza in primul rand impotriva florei gram-negative.

Fluorochinolonele moderne din generațiile II, III și IV, spre deosebire de predecesorii lor, sunt antibiotice cu cel mai larg, ca să spunem așa, spectru de acțiune. Acestea includ Levofloxacin, Moxifloxacin, Gatifloxacin și alte medicamente active împotriva:

Rețineți că toate fluorochinolonele, fără excepție, sunt contraindicate pentru utilizare la copiii sub 18 ani. Acest lucru se datorează capacității antibioticelor din acest grup de a perturba sinteza peptidoglicanului, o substanță inclusă în structura tendonului. Prin urmare, administrarea de fluorochinolone la copii este asociată cu riscul de modificări ale țesutului cartilajului.

Fluorochinolona II generația, Levofloxacin este prescris pentru infecții ale tractului respirator - pneumonie, bronșită, organe ORL - sinuzită, otită, precum și boli ale tractului urinar, tractului reproducător, inclusiv chlamydia urogenitală, infecții ale pielii (furunculoză) și țesuturilor moi ( ateroame, abcese).

Levofloxacina este prescrisă 500 mg pe zi timp de șapte, mai rar - 10 zile. În cazurile severe, antibioticul se administrează intravenos.

In rusa piata farmaceutica Multe medicamente care conțin lomefloxacină sunt înregistrate. Produsul original - marca - este Tavanik german. Genericele sale includ Levofloxacin Teva, Levolet, Glevo, Flexil, Ecolevid, Hyleflox și alte medicamente.

Moxifloxacină

Moxifloxacina este un antibiotic fluorochinolone cu spectru larg de a treia generație foarte activ, indicat pentru infecții ale organelor ORL, tractului respirator, pielii, țesuturilor moi și infecțiilor postoperatorii. Medicamentul este prescris în tablete de 400 mg o dată pe zi. Cursul tratamentului variază de la 7 la 10 zile.

Medicamentul original al moxifloxacinei, care este cel mai des utilizat, este Avelox produs de Bayer. Există foarte puține medicamente generice ale Avelox și este destul de dificil să le găsești în farmacii. Moxifloxacina face parte din picăturile oftalmice Vigamox, indicate pentru procesele inflamatorii infecțioase ale conjunctivei oculare și alte boli.

Gatifloxacină

Medicamentul de cea mai recentă generație a IV-a de fluorochinolone este prescris pentru boli severe, inclusiv a tractului respirator dobândite în spital, patologii oftalmologice, infecții ale organelor ORL și tractului urogenital. Efectul antibacterian al Gatifloxacinei se aplică și agenților patogeni cu transmitere sexuală.

Gatifloxacina este prescrisă 200 sau 400 mg pe zi o dată.

Majoritatea medicamentelor care conțin gatifloxacină sunt produse de companii indiene. Cel mai adesea în farmacii găsești Tabris, Gaflox, Gatispan.

Aminoglicozide: antibiotice esențiale

Aminoglicozidele cuprind un grup de medicamente antibacteriene care au proprietăți similare ca structură și, desigur, spectrul de acțiune. Aminoglicozidele inhibă sinteza proteinelor în microbi, exercitând un efect bactericid pronunțat împotriva microorganismelor sensibile.

Primul aminoglicozid este un antibiotic natural izolat în timpul celui de-al Doilea Război Mondial. În mod surprinzător, ftiziologia modernă nu se poate lipsi de aceeași streptomicină, care a fost descoperită în 1943 - antibioticul este acum utilizat pe scară largă în ftiziologie pentru a trata tuberculoza.

Toate cele patru generații de aminoglicozide, care au fost izolate și sintetizate treptat pe parcursul a mai bine de jumătate de secol, au un spectru la fel de larg de acțiune antibacteriană. Antibioticele din acest grup acționează asupra:

  • coci gram-pozitivi: streptococi și stafilococi;
  • microorganisme gram-negative: Escherichia coli, Klebsiella, Salmonella, Shigella, Moraxella, Pseudomonas și altele.

Aminoglicozidele din diferite generații au unele caracteristici individuale, pe care vom încerca să le urmărim folosind exemple de medicamente specifice.

Cea mai veche aminoglicozidă cu acțiune largă din prima generație în injecții, care se distinge prin activitatea sa antibacteriană ridicată împotriva mycobacterium tuberculosis. Indicațiile pentru utilizarea streptomicinei sunt tuberculoza primară de orice localizare, ciuma, bruceloza și tularemia. Antibioticul se administrează intramuscular, intratraheal și, de asemenea, intracavernos.

Un antibiotic foarte controversat din a doua generație, care cade treptat în uitare, este gentamicina. Ca și alte aminoglicozide din a doua generație și cele mai vechi, Gentamicina este activă împotriva Pseudomonas aeruginosa. Antibioticul există în trei forme: injectabil, extern sub formă de unguente și local ( picaturi de ochi).

Interesant, spre deosebire de marea majoritate a antibioticelor, gentamicina își păstrează perfect proprietățile în formă dizolvată. Prin urmare, forma de injectare a medicamentului este o soluție gata preparată în fiole.

Gentamicina este utilizată pentru boli infecțioase și inflamatorii ale tractului biliar - colecistită, colangită, tract urinar - cistita, pielonefrită, precum și pentru infecții ale pielii și țesuturilor moi. În practica oftalmologică, picăturile oftalmice cu Gentamicină sunt prescrise pentru blefarită, conjunctivită, keratită și alte leziuni infecțioase ale ochiului.

Motivul unei atitudini precaute față de gentamicina este datele despre efecte secundare antibiotic, în special ototoxicitate. În ultimii ani, s-au obținut suficiente dovezi cu privire la deficiența de auz datorată terapiei cu Gentamicină. Există chiar și cazuri de surditate completă care s-au dezvoltat din cauza administrării unui antibiotic. Pericolul este că, de regulă, efectul ototoxic al gentamicinei este ireversibil, adică auzul nu este restabilit după întreruperea antibioticului.

Pe baza acestei tendințe triste, majoritatea medicilor preferă să opteze pentru alte aminoglicozide, mai sigure.

Amikacin

O alternativă excelentă la Gentamicină este antibioticul cu spectru larg de a treia generație Amikacin, care este produs sub formă de pulbere pentru prepararea unei soluții injectabile. Indicațiile pentru utilizarea Amikacin includ peritonită, meningită, endocardită, sepsis, pneumonie și alte boli infecțioase severe.

Amfenicoli: hai să vorbim despre Levomicetina veche

Principalul reprezentant al grupului amfenicol este antibiotic natural cloramfenicol cu ​​spectru larg, care este cunoscut de aproape toți compatrioții noștri sub numele de Levomycetin. Medicamentul este un izomer structural levogitor al cloramfenicolului (de unde prefixul „stânga”).

Spectrul de acțiune al Levomicetinei acoperă:

  • coci gram-pozitivi: stafilococi și streptococi;
  • bacterii gram-negative: agenți patogeni de gonoree, Escherichia coli și Haemophilus influenzae, Salmonella, Shigella, Yersinia, Proteus, Rickettsia.

În plus, levomicetina este activă împotriva spirochetelor și chiar a unor viruși mari.

Indicațiile pentru utilizarea Levomycetin includ febră tifoidă și paratifoidă, dizenterie, bruceloză, tuse convulsivă, tifos și diverse infecții intestinale.

Formele externe de Levomycetin (unguent) sunt prescrise pentru bolile purulente ale pielii, ulcere trofice. Astfel, în Rusia, un unguent care conține Levomycetin, care este produs sub numele de Levomekol, este foarte popular.

În plus, levomicetina este utilizată în oftalmologie pentru bolile inflamatorii oculare.

Un curs de tratament cu Levomycetin sau Cum să vă dăunezi organismului?

Levomicetina este un antibiotic intestinal accesibil, eficient și, prin urmare, cu spectru larg, iubit de mulți. Atât de iubit încât poți întâlni adesea un pacient într-o farmacie, cumpărând aceleași pastile anti-diaree și lăudându-le eficacitatea. Desigur: am luat două-trei comprimate - și problemele au dispărut. Tocmai în această abordare a tratamentului cu Levomycetin se ascunde pericolul.

Nu trebuie să uităm că Levomicetina este un antibiotic care trebuie luat într-o cură. Știm că, de exemplu, antibioticul Amoxicilină nu trebuie luat mai puțin de cinci zile, dar consumând două comprimate de Levomycetin, reușim să uităm complet de originea antibacteriană a medicamentului. Ce se întâmplă cu bacteriile în acest caz?

Este simplu: cele mai slabe enterobacterii, desigur, mor după două sau trei doze de Levomicetin. Diareea încetează, iar noi, dând slavă puterii pastilelor amare, uităm de necazuri. Între timp, microorganismele puternice și persistente supraviețuiesc și își continuă funcțiile vitale. Adesea ca agenți patogeni oportuniști, care devin mai activi la cea mai mică scădere a imunității și ne arată unde hibernează racii. Atunci levomicetina poate să nu mai facă față microbilor selectați.

Pentru a preveni acest lucru, ar trebui să urmați cursul recomandat de terapie cu antibiotice. Pentru tratamentul infecțiilor intestinale acute, medicamentul este administrat într-o doză de 500 mg de trei până la patru ori pe zi timp de cel puțin o săptămână. Dacă nu sunteți dispus să vă conformați suficient curs intensiv, este mai bine să acordați preferință altor medicamente antimicrobiene, de exemplu, derivații de nitrofuran.

Carbapenemi: antibiotice de rezervă

De regulă, întâlnim carbapenemi extrem de rar sau deloc. Și acest lucru este minunat - la urma urmei, aceste antibiotice sunt indicate pentru tratamentul infecțiilor severe ale spitalelor care amenință viața. Spectrul de acțiune al carabapenemelor include majoritatea tulpinilor patologice existente, inclusiv cele rezistente.

Antibioticele din acest grup includ:

  • Meropenem. Cel mai comun carbapenem, care este produs sub denumirile comerciale Meronem, Meropenem, Cyronem, Jenem etc.;
  • Ertapenem, nume comercial Invanz;
  • Imipenem.

Carbapenemele se administrează numai intravenos, intravenos în perfuzie și bolus, adică folosind un dozator special.

Terapia cu antibiotice: regula de aur a siguranței

La sfârșitul excursiei noastre în lumea antibioticelor cu spectru larg, nu putem ignora cel mai important aspect, pe care se bazează siguranța medicamentelor și, în cele din urmă, sănătatea noastră. Fiecare pacient – ​​actual sau potențial – ar trebui să știe și să-și amintească că dreptul de a prescrie antibiotice aparține exclusiv medicului.

Indiferent de câte cunoștințe crezi că ai în domeniul medicinei, nu ar trebui să cedezi tentației de a te „trata singur”. Mai mult, nu ar trebui să vă bazați pe abilitățile farmaceutice ipotetice ale vecinilor, prietenilor și colegilor.

Numai doctor bun. Aveți încredere în cunoștințele și experiența unui mare specialist, iar acest lucru vă va ajuta să vă mențineți sănătatea pentru mulți ani.

Antibioticele sunt un grup de medicamente care au un efect dăunător sau distructiv asupra bacteriilor care provoacă boli infecțioase. La fel de agenți antivirali Acest tip de medicament nu este utilizat. În funcție de capacitatea de a distruge sau de a inhiba anumite microorganisme, există diferite grupuri de antibiotice. În plus, acest tip de medicament poate fi clasificat în funcție de originea sa, natura efectului său asupra celulelor bacteriene și unele alte caracteristici.

descriere generala

Antibioticele aparțin grupului de medicamente biologice antiseptice. Sunt produse reziduale ale ciupercilor mucegăite și radiante, precum și ale unor tipuri de bacterii. În prezent, sunt cunoscute peste 6.000 de antibiotice naturale. În plus, există zeci de mii de sintetice și semi-sintetice. Dar doar aproximativ 50 de astfel de medicamente sunt folosite în practică.

Grupurile principale

Toate aceste medicamente care există în prezent sunt împărțite în trei grupuri mari:

  • antibacterian;
  • antifungic;
  • antitumoral.

În plus, în funcție de direcția de acțiune, acest tip de medicament este împărțit în:

  • activ împotriva bacteriilor gram-pozitive;
  • anti-tuberculoză;
  • activ împotriva bacteriilor gram-pozitive și gram-negative;
  • antifungic;
  • distrugerea helminților;
  • antitumoral.

Clasificarea după tipul de efect asupra celulelor microbiene

În acest sens, există două grupe principale de antibiotice:

  • Bacteriostatic. Medicamentele de acest tip suprimă dezvoltarea și reproducerea bacteriilor.
  • Bactericid. Atunci când se utilizează medicamente din acest grup, microorganismele existente sunt distruse.

Tipuri după compoziția chimică

Clasificarea antibioticelor în grupuri în acest caz este următoarea:

  • Penicilinele. Acesta este cel mai vechi grup cu care, de fapt, a început dezvoltarea acestei direcții de tratament medicamentos.
  • Cefalosporine. Acest grup folosit foarte larg și diferă grad înalt rezistenta la actiunea distructiva a β-lactamazelor. Acesta este numele dat enzimelor speciale secretate de microorganismele patogene.
  • Macrolide. Acestea sunt cele mai sigure și destul de eficiente antibiotice.
  • Tetracicline. Aceste medicamente sunt utilizate în principal pentru a trata sistemul respirator și tractul urinar.
  • Aminoglicozide. Au un spectru foarte larg de acțiune.
  • Fluorochinolone. Preparate bactericide slab toxice.

Aceste antibiotice sunt utilizate în Medicină modernă mai des. Pe langa ele, mai sunt si altele: glicopeptide, poliene etc.

Antibiotice din grupa penicilinei

Medicamentele de acest tip sunt baza fundamentală a absolut orice tratament antimicrobian. La începutul secolului trecut, nimeni nu știa despre antibiotice. În 1929, englezul A. Fleming a descoperit primul astfel de medicament - penicilina. Principiul de acțiune al medicamentelor din acest grup se bazează pe suprimarea sintezei proteinelor în pereții celulari ai agentului patogen.

Pe în prezent Există doar trei grupe principale de antibiotice peniciline:

  • biosintetice;
  • semi sintetic;
  • spectru larg semisintetic.

Primul tip este utilizat în principal pentru tratamentul bolilor cauzate de stafilococi, streptococi, meningococi etc. Astfel de antibiotice pot fi prescrise, de exemplu, pentru boli precum pneumonia, leziunile infecțioase ale pielii, gonoreea, sifilisul, gangrena gazoasă etc.

Antibioticele semisintetice din grupul penicilinei sunt cel mai adesea utilizate pentru a trata infecțiile stafilococice severe. Astfel de medicamente sunt mai puțin active împotriva anumitor tipuri de bacterii (de exemplu, gonococi și meningococi) decât cele biosintetice. Prin urmare, înainte de numirea lor, de obicei sunt efectuate proceduri precum izolarea și identificarea precisă a agentului patogen.

Penicilinele semisintetice cu spectru larg sunt de obicei folosite dacă antibioticele tradiționale (cloramfenicol, tetraciclină etc.) nu ajută pacientul. Această varietate include, de exemplu, grupul de antibiotice amoxicilinei destul de frecvent utilizat.

Patru generații de peniciline

În miere În practică astăzi, sunt utilizate patru tipuri de antibiotice din grupul penicilinei:

  • Prima generație este medicamentele de origine naturală. Acest tip de medicament are o gamă foarte restrânsă de aplicații și nu este foarte rezistent la penicilinaze (β-lactamaze).
  • A doua și a treia generație sunt antibiotice care sunt mult mai puțin sensibile la enzimele distructive ale bacteriilor și, prin urmare, mai eficiente. Tratamentul cu utilizarea lor poate avea loc într-un timp destul de scurt.
  • A patra generație include antibiotice peniciline cu spectru larg.

Cele mai cunoscute peniciline sunt semi droguri sintetice„Ampicilină”, „Carbenicilină”, „Azocilină”, precum și „Benzylpenicilină” biosintetică și formele sale durabile (biciline).

Efecte secundare

Deși antibioticele din acest grup sunt medicamente cu toxicitate scăzută, ele, împreună cu efectele lor benefice, pot avea și un efect negativ asupra organismului uman. Efectele secundare la utilizarea lor sunt următoarele:

  • mâncărime și erupții cutanate;
  • reactii alergice;
  • disbacterioză;
  • greață și diaree;
  • stomatita.

Penicilinele nu pot fi utilizate simultan cu antibiotice din alt grup - macrolide.

Grupul de antibiotice amoxicilinei

Acest tip de medicament antimicrobian aparține penicilinelor și este utilizat pentru a trata bolile cauzate de infecția cu bacterii gram-pozitive și gram-negative. Astfel de medicamente pot fi utilizate pentru a trata atât copiii, cât și adulții. Cel mai adesea, antibioticele pe bază de amoxicilină sunt prescrise pentru infecții ale tractului respirator și diferite boli gastrointestinale. De asemenea, sunt luate pentru boli ale sistemului genito-urinar.

Grupul de antibiotice amoxicilină este, de asemenea, utilizat pentru diferite infecții ale țesuturilor moi și ale pielii. Aceste medicamente pot provoca aceleași reacții adverse ca și alte peniciline.

Grupul de cefalosporine

Acțiunea medicamentelor din acest grup este și bacteriostatică. Avantajul lor față de peniciline este rezistența lor bună la β-lactamaze. Antibioticele grupului de cefalosporine sunt clasificate în două grupe principale:

  • administrat parenteral (ocolind tractul gastrointestinal);
  • luată pe cale orală.

În plus, cefalosporinele sunt clasificate în:

  • Medicamente de prima generație. Au un spectru îngust de acțiune și practic nu au niciun efect asupra bacteriilor gram-negative. Mai mult, astfel de medicamente sunt utilizate cu succes în tratamentul bolilor cauzate de streptococi.
  • Cefalosporine de a doua generație. Mai eficient împotriva bacteriilor gram-negative. Sunt activi împotriva stafilococilor și streptococilor, dar practic nu au niciun efect asupra eterococilor.
  • Medicamente de generația a treia și a patra. Acest grup de medicamente este foarte rezistent la acțiunea β-lactamazelor.

Principalul dezavantaj al medicamentelor precum antibioticele cefalosporine este că, atunci când sunt administrate pe cale orală, sunt foarte iritante pentru mucoasa gastrointestinală (cu excepția medicamentului Cephalexin). Avantajul medicamentelor de acest tip este că cantitatea de indusă efecte secundare. Medicamentele cel mai des utilizate în practica medicală sunt Cefalotin și Cefazolin.

Efectele negative ale cefalosporinelor asupra organismului

Efectele secundare care apar uneori la administrarea de antibiotice din această serie includ:

  • efecte negative asupra rinichilor;
  • încălcarea funcției hematopoietice;
  • diverse tipuri de alergii;
  • impact negativ asupra tractului gastro-intestinal.

Antibioticele din grupa macrolidelor

Printre altele, antibioticele sunt clasificate în funcție de gradul de selectivitate a acțiunii. Unele sunt capabile să afecteze negativ doar celulele agentului patogen, fără a afecta țesutul uman în vreun fel. Altele pot avea un efect toxic asupra corpului pacientului. Medicamentele din grupa macrolidelor sunt considerate cele mai sigure în acest sens.

Există două grupe principale de antibiotice din acest soi:

  • natural;
  • semi sintetic.

Principalele avantaje ale macrolidelor includ cea mai mare eficiență a efectelor bacteriostatice. Sunt activi în special împotriva stafilococilor și streptococilor. Printre altele, macrolidele nu afectează negativ mucoasa gastrointestinală și, prin urmare, sunt adesea disponibile în tablete. Toate antibioticele afectează sistemul imunitar uman într-o măsură sau alta. Unele tipuri sunt deprimante, altele sunt benefice. Antibioticele din grupul macrolidelor au un efect imunomodulator pozitiv asupra corpului pacientului.

Macrolidele populare sunt Azitromicină, Sumamed, Eritromicină, Fuzidin etc.

Antibiotice din grupa tetraciclinei

Medicamentele de acest tip au fost descoperite pentru prima dată în anii 40 ai secolului trecut. Primul medicament tetraciclină a fost izolat de B. Duggar în 1945. Se numea „Clortetraciclină” și era mai puțin toxică decât alte antibiotice existente la acea vreme. În plus, s-a dovedit a fi foarte eficient în ceea ce privește influențarea agenților patogeni ai unui număr mare de boli foarte periculoase (de exemplu, tifoidă).

Tetraciclinele sunt considerate ceva mai puțin toxice decât penicilinele, dar au efecte mai negative asupra organismului decât antibioticele macrolide. Prin urmare, în acest moment, acestea sunt înlocuite în mod activ de acestea din urmă.

Astăzi, medicamentul „Clortetraciclină”, descoperit în secolul trecut, destul de ciudat, este folosit foarte activ nu în medicină, ci în agricultură. Faptul este că acest medicament poate accelera creșterea animalelor luând-o de aproape două ori. Substanța are un astfel de efect, deoarece atunci când intră în intestinele animalului, începe să interacționeze activ cu microflora prezentă în acesta.

În plus față de medicamentul „Tetraciclină” în sine, în practica medicală sunt adesea utilizate medicamente precum „Metaciclină”, „Vibramicină”, „Doxycycline”, etc.

Efecte secundare cauzate de antibioticele tetracicline

Refuz aplicare largă Utilizarea medicamentelor de acest tip în medicină se datorează în primul rând faptului că pot avea efecte nu numai benefice, ci și negative asupra corpului uman. De exemplu, când utilizare pe termen lung, antibioticele tetracicline pot interfera cu dezvoltarea oaselor și a dinților la copii. În plus, prin interacțiunea cu microflora intestinului uman (dacă sunt utilizate incorect), astfel de medicamente provoacă adesea dezvoltarea bolilor fungice. Unii cercetători susțin chiar că tetraciclinele pot avea un efect deprimant asupra sistemului reproducător masculin.

Antibiotice din grupa aminoglicozidelor

Preparatele de acest tip au un efect bactericid asupra agentului patogen. Aminoglicozidele, precum penicilinele și tetraciclinele, sunt unul dintre cele mai vechi grupuri de antibiotice. Au fost deschise în 1943. În anii următori, medicamentele de acest tip, în special streptomicina, au fost utilizate pe scară largă pentru a trata tuberculoza. În special, aminoglicozidele sunt eficiente împotriva bacteriilor aerobe gram-negative și a stafilococilor. Printre altele, unele medicamente din această serie sunt active și împotriva protozoarelor. Deoarece aminoglicozidele sunt mult mai toxice decât alte antibiotice, ele sunt prescrise numai pentru boli severe. Sunt eficiente, de exemplu, în sepsis, tuberculoză, forme severe paranefrită, abcese abdominale etc.

Foarte des, medicii prescriu aminoglicozide precum neomicina, kanamicina, gentamicina etc.

Medicamente din grupa fluorochinolonelor

Majoritatea medicamentelor acestui tip de antibiotic au un efect bactericid asupra agentului patogen. Avantajele lor includ, în primul rând, cea mai mare activitate împotriva unui număr mare de microbi. Ca și aminoglicozidele, fluorochinolonele pot fi folosite pentru a trata boli grave. Cu toate acestea, ele nu au un efect atât de negativ asupra corpului uman ca primele. Există antibiotice din grupul fluorochinolonelor:

  • Prima generatie. Acest soi folosit în principal pentru tratament internat bolnav. Fluorochinolonele de prima generație sunt utilizate pentru infecții ale ficatului, căilor biliare, pneumoniei etc.
  • A doua generație. Aceste medicamente, spre deosebire de primele, sunt foarte active împotriva bacteriilor gram-pozitive. Prin urmare, sunt prescrise și pentru tratament fără spitalizare. Fluorochinolonele de a doua generație sunt utilizate pe scară largă pentru bolile cu transmitere sexuală.

Medicamentele populare din acest grup sunt Norfloxacin, Levofloxacin, Gemifloxacin etc.

Așadar, am aflat cărei grupe aparțin antibioticele și ne-am dat seama exact cum sunt clasificate. Deoarece cele mai multe dintre aceste medicamente pot provoca reacții adverse, acestea ar trebui să fie utilizate numai conform indicațiilor medicului dumneavoastră.

Antibioticele sunt produse metabolice ale microorganismelor care suprimă activitatea altor microbi. Antibioticele naturale, precum și derivații lor semisintetici și analogii sintetici, sunt utilizate ca medicamente, care au capacitatea de a suprima agenții patogeni ai diferitelor boli din corpul uman.

Pe baza structurii lor chimice, antibioticele sunt împărțite în mai multe grupuri:

A. Antibiotice beta-lactamice.

1. Peniciline.

a) Peniciline naturale: benzilpenicilina si sarurile sale, fenoximetil penicilina.

b) Peniciline semisintetice:

Rezistent la penicilinază cu activitate primară împotriva stafilococilor: oxacilină, cloxacilină, flucloxacilină;

Cu activitate preferenţială împotriva bacteriilor gram-negative (amidinopeniciline); amdinocilină (mecilinam), acidocilină;

Spectru larg (aminopeniciline): ampicilină, amoxicilină, pivampicilină;

Spectru larg de acțiune, în special foarte activ împotriva Pseudomonas aeruginosa și a altor bacterii gram-negative (carboxi- și uree-dopeniciline): carbenicilină, tikarishin, azlocilină, mezlocilină, piperacilină.

2. Cefalosporine:

a) prima generație: cefaloridină, cefazolină etc.;

b) generația a doua: cefamandol, cefuroximă etc.;

c) generația a treia: cefotaximă, ceftazidimă etc.;

d) generația a patra: cefpirom, cefepime etc.

3. Monobactami: aztreonam.

4. Carbapeneme: imipenem, meronem, tienam, primaxină. B. Fosfomicină.

B. Macrolide:

a) prima generație: eritromicină, oleandomicină;

b) generația a doua: spiramicină (Rovamycin), roxitromicină (Rulid), claritromicină (Klacid) etc.;

c) a treia generație: azitromicină (sumamed). G. Lincosamide: lincomicină, clindamicină. D. Fuzidin.

E. Aminoglicozide:

a) prima generație: streptomicina, monomicină, kanamicina;

b) a doua generatie: gentamicina;

c) generația a treia: tobramicină, sisomicina, amikacina, netilmicină;

d) generația a patra: isepamicină. J. Levomicetina.

3. Tetracicline: a) naturale: tetraciclina, oxitetraciclina, clortetraciclina; b) semisintetice: metaciclina, doxiciclina, minociclina, morfociclina.

ȘI. Rifamicinele: rifocină, rifamidă, rifampicină.

LA. Antibiotice glicopeptidice: vancomicină, teicoplanină.

L. Ristomicina.

M. Polimixine: polimixină B, polimixină E, polimixină M.

H. Gramicidină.

DESPRE. Antibiotice poliene: nistatină, levorină, amfotericină B.

Pe baza naturii acțiunii lor antimicrobiene, antibioticele sunt împărțite în bactericide și bacteriostatice. Medicamentele bactericide care provoacă moartea microorganismelor includ penicilinele, cefalosporinele, aminoglicozidele, polimixinele etc. Astfel de medicamente pot da un efect terapeutic rapid în infecțiile severe, care este deosebit de important la copiii mici. Utilizarea lor este mai rar însoțită de recidive ale bolilor și cazuri de transport. Antibioticele bacteriostatice includ tetracicline, cloramfenicol, macrolide etc. Aceste medicamente, prin perturbarea sintezei proteinelor, inhibă diviziunea microorganismelor. Ele sunt de obicei destul de eficiente pentru boli moderat severe.

Antibioticele sunt capabile să inhibe procesele biochimice care apar în microorganisme. În funcție de mecanismul lor de acțiune, ele sunt împărțite în următoarele grupuri:

1. Inhibitori ai sintezei peretelui microbian sau componentelor acestuia în timpul mitozei: peniciline, cefalosporine, carbapeneme, monobactami, antibiotice glicopeptidice, ristomicina, fosfomicină, cicloserina.

2. Antibiotice care perturbă structura și funcția membranelor citoplasmatice: polimixine, aminoglicozide, antibiotice poliene, gramicidină, antibiotice glicopeptidice.

3. Inhibitori ai sintezei ARN la nivelul ARN polimerazei: rifamicine.

4. Inhibitori ai sintezei ARN la nivel ribozom: cloramfenicol, macrolide (eritromicină, oleandomicină etc.), lincomicină, clindamicină, fusidină, tetracicline, aminoglicozide (kanamicină, gentamicină etc.), antibiotice glicopeptidice.

În plus, important în mecanismul de acțiune al antibioticelor individuale, în special penicilinelor, este efectul inhibitor al acestora asupra aderenței microorganismelor la membranele celulare.

Mecanismul de acțiune al antibioticelor determină în mare măsură tipul de efecte pe care le provoacă. Astfel, antibioticele care perturbă sinteza peretelui microbian sau funcția membranelor citoplasmatice sunt medicamente bactericide; antibiotice care inhibă sinteza acizi nucleiciși proteinele, de obicei acționează bacteriostatic. Cunoașterea mecanismului de acțiune al antibioticelor este necesară pentru selectarea corectă a acestora, determinarea duratei tratamentului, selectarea combinațiilor eficiente de medicamente etc.

Pentru a oferi terapie etiotropă, este necesar să se țină cont de sensibilitatea agenților patogeni la antibiotice. Sensibilitatea naturală la acestea se datorează proprietăților biologice ale microorganismelor, mecanismului de acțiune al antibioticelor și altor factori. Există antibiotice cu spectru îngust și larg. Antibioticele cu spectru îngust includ medicamente care suprimă predominant bacteriile gram-pozitive sau gram-negative: unele peniciline (benzilpenicilină, oxacilină, acido-cilină, aztreonam, ristomicina, fusidin, novobiocina, bacitracină, vancomicina, monobactamii (aztreonam) au și polimixinele B. un spectru îngust.E, M, bacterii gram-negative inhibitoare, precum și antibiotice antifungice nistatina, levorina, amfotericina B, amfoglucamină, micoheptina, griseofulvină.

Antibioticele cu spectru larg includ medicamente care afectează atât bacteriile gram-pozitive, cât și gram-negative: o serie de peniciline semisintetice (ampicilină, amoxicilină, carbenicilină); cefalosporine, în special a treia și a patra generație; carbapeneme (imipenem, meronem, tienam); cloramfenicol; tetracicline; aminoglicozide; rifamicinelor. Unele dintre aceste antibiotice acționează și asupra rickettsiei, chlamidiei, micobacteriilor etc.

Când se identifică agentul cauzal al unei boli infecțioase și sensibilitatea acestuia la antibiotice, este de preferat să se utilizeze medicamente cu un spectru îngust de acțiune. Antibioticele cu spectru larg sunt prescrise pentru boli severe și infecții mixte.

Antibioticele includ medicamente care se acumulează în interiorul celulelor (raportul dintre concentrațiile intra și extracelulare este mai mare de 10). Acestea includ macrolide, în special cele noi (azitromicină, roxitromicină, spiramicină), carbapenemi și clindamicină. Rifampicina, cloramfenicolul, tetraciclinele, lincomicina, vancomicina, teicoplanina, fosfomicina pătrund bine în celule (raportul concentrațiilor intra și extracelulare este de la 1 la 10). Penicilinele, cefalosporinele, aminoglicozidele pătrund slab în celule (raportul dintre concentrațiile intra și extracelulare este mai mic de 1). De asemenea, polimixinele nu pătrund în celule.

În procesul de utilizare a antibioticelor, microorganismele pot dezvolta rezistență la acestea. La peniciline, cefa osporine, monobactami, carba-penemi, cloramfenicol, tetracicline, glicopeptide, ristomicina, fosfomicină, lincosamide, rezistența se dezvoltă lent și în paralel scade efectul terapeutic al medicamentelor. Rezistența la aminoglicozide, macrolide, rifamicine, polimixine și fusidină se dezvoltă foarte rapid, uneori în timpul tratamentului unui pacient.

CARACTERISTICILE GRUPURILOR SEPARATE DE ANTIBIOTICE

Penicilinele. Conform structurii lor chimice, aceste antibiotice sunt derivați ai acidului 6-aminopenicilanic (6-APA) care conțin diverși substituenți (R) în grupa amino.

Mecanismul acțiunii antimicrobiene a penicilinelor este acela de a perturba formarea peretelui celular din fragmente de mureină pre-sintetizate. Există peniciline naturale: benzilpenicilină (sub formă de săruri de sodiu, potasiu, novocaină), biciline, fenoximetilpenicilină; peniciline semisintetice: oxacilină, cloxacilină, ampicilină (pentrexil), amoxicilină, carbenicilină, carfecilină, piperacilină, mezlocilină, azlocilină etc.

Benzilpenicilina dă un efect terapeutic clar în tratamentul bolilor cauzate de pneumococi, stafilococi, streptococi de grup hemolitic A, meningococi, gonococi, spirochete pallidum, corynobacterii, bacil antrax și alte microorganisme. Multe tulpini de microbi, în special stafilococii, sunt rezistente la benzilpenicilină, deoarece produc o enzimă (3-lactamaza, care inactivează antibioticul.

Benzilpenicilina se administreaza de obicei intramuscular, in situatii critice intravenos (numai sare de sodiu). Dozele variază foarte mult de la 30.000-50.000 EDUkg/zi) la 1.000.000 EDU/kg/zi) în funcție de agentul patogen, severitatea și localizarea procesului infecțios.

Concentrația terapeutică în plasma sanguină apare în 15 minute după administrarea intramusculară și rămâne acolo timp de 3-4 ore.Benzilpenicilina pătrunde bine în mucoase și plămâni. Intră puțin în lichidul cefalorahidian, miocard, oase, pleural, lichid sinovial, în lumenul bronhiilor și în uter. Pentru meningită este posibilă administrarea endolombară de sare de sodiu de benzilpenicilină. Medicamentul poate fi administrat în cavități, endobronșic, endolimfatic. Se găsește în concentrații mari în bilă și urină. La copiii cu vârsta sub o lună, eliminarea benzilpenicilinei are loc mai lent decât la adulți. Aceasta determină frecvența de administrare a medicamentului: în prima săptămână de viață de 2 ori pe zi, apoi de 3-4 ori, iar după o lună, ca la adulți, de 5-6 ori pe zi.

Când se tratează infecții care necesită terapie cu antibiotice pe termen lung și nu au curs acut(infecție streptococică focală, sifilis), preparatele cu benzilpenicilină cu acțiune prelungită sunt utilizate pentru prevenirea exacerbărilor reumatismului: sare de novocaină, ? bicilinele 1, 3, 5. Aceste medicamente nu diferă în spectrul de acțiune antimicrobiană de sărurile de sodiu și potasiu ale benzilpenicilinei, pot fi utilizate la copiii cu vârsta peste 1 an. Toate penicilinele cu acțiune prelungită se administrează numai intramuscular sub formă de suspensie. După o singură injecție de sare de novocaină, concentrația terapeutică a benzilpenicilinei în sânge rămâne până la 12 ore.Bicilin-5 se administrează o dată la 2 săptămâni. Injecțiile cu bicilină-1 și bicilină-3 se efectuează o dată pe săptămână. Bicilinele sunt utilizate în principal pentru a preveni recidivele reumatismului.

Fenoximetilpenicilina- o formă de penicilină rezistentă la acid, utilizată pe cale orală pe stomacul gol de 4-6 ori pe zi pentru tratamentul bolilor infecțioase ușoare. Spectrul său de acțiune este aproape același cu cel al benzilpenicilinei.

Ospen (bimepen) benzatină fenoximetilpenicilină este absorbit lent din tractul gastrointestinal și menține concentrațiile terapeutice în sânge pentru o lungă perioadă de timp. Se prescrie sub formă de sirop de 3 ori pe zi.

Oxacilină, cloceacilină, flucloxacilină- peniciline semisintetice, utilizate în principal în tratamentul bolilor cauzate de stafilococi, inclusiv a celor rezistente la benzilpenicilină. Oxacilina este capabilă să inhibe (3-lactamaza stafilococilor și să sporească efectul altor peniciline, de exemplu ampicilina (un medicament combinat de oxacilină cu ampicilină - ampiox). Pentru bolile cauzate de alte microorganisme sensibile la benzilpenicilină (meningococi, gonococi, pneumococi, streptococi, spirochete etc.), aceste antibiotice sunt practic rar utilizate din cauza lipsei unui efect pozitiv.

Oxacilina, cloxacilina, flucloxacilina sunt bine absorbite din tractul gastrointestinal. În plasma sanguină, aceste medicamente sunt legate de proteine ​​și nu pătrund bine în țesuturi. Aceste antibiotice pot fi administrate intramuscular (la fiecare 4-6 ore) si intravenos prin jet sau picurare.

Amidinopeniciline - amdinocilina (mecillinam) este un antibiotic cu spectru îngust, inactiv împotriva bacteriilor gram-pozitive, dar suprimă eficient bacteriile gram-negative (Escherichia coli, Shigella, Salmonella, Klebsiella). Pseudomonas aeruginosa, Proteus și bacteriile gram-negative nefermentante sunt de obicei rezistente la amdinocilină. Particularitatea acestui antibiotic este că interacționează activ cu PSB-2 (proteina de legare a penicilinei), în timp ce majoritatea celorlalte (antibiotice 3-lactamice) interacționează cu PSB-1 ​​​​și PSB-3. Prin urmare, poate fi un sinergic cu alte peniciline, precum și cefalosporine.Medicamentul se administrează parenteral și pătrunde în celule de multe ori mai bine decât ampicilina și carbenicilina.Antibioticul este eficient în special împotriva infecțiilor tractului urinar.Un derivat eteric al medicamentului pivamdinocilină a fost sintetizat pentru uz enteral. .

Penicilinele semisintetice cu spectru larg - ampicilină, amoxicilină - sunt de cea mai mare importanță în tratamentul bolilor cauzate de Haemophilus influenzae, gonococi, meningococi, unele tipuri de Proteus, salmonella și, în plus, agenți patogeni ai listeriozei și enterococilor. Aceste antibiotice sunt eficiente și pentru tratamentul proceselor infecțioase cauzate de microflora mixtă (gram-pozitivă și gram-negativă). Ampicilina și amoxicilina pot fi administrate oral, de exemplu, în tratamentul infecțiilor tractului gastrointestinal, tractului urinar și otitei medii. Ampicilina care nu este absorbită din tractul gastrointestinal provoacă iritarea membranelor mucoase, ducând la vărsături, diaree și iritații ale pielii din jur la un procent semnificativ de copii. anus. Amoxicilina diferă de ampicilină printr-o absorbție mai bună, așa că poate fi prescrisă pe cale orală nu numai pentru infecții ușoare, ci și pentru infecții moderate. Amoxicilina este mai puțin iritantă pentru membranele mucoase ale tractului gastrointestinal și este mai puțin probabil să provoace vărsături și diaree. Pentru bolile severe care necesită crearea unei concentrații mari de antibiotic în sânge, aceste medicamente sunt administrate parenteral.

Carboxipeniciline- carbenicilina, ticarcilina au un spectru și mai mare de acțiune antimicrobiană decât ampicilina și diferă de aceasta prin capacitatea suplimentară de a suprima Pseudomonas aeruginosa, tulpinile indol-pozitive ale Proteus și bacteroides. Utilizarea lor principală este bolile cauzate de acești agenți patogeni. Carbenicilina și ticarcilina sunt absorbite foarte slab din tractul gastrointestinal, deci sunt utilizate numai parenteral (carbenicilina intramuscular și intravenos, ticarcilina intravenos). Carfecilina este un ester fenilic al carbenicilinei. Este bine absorbit din tractul gastrointestinal, după care carbenicilina este eliberată din acesta. Carboxipenicilinele, în comparație cu ampicilina, pătrund mai rău în țesuturi, cavități seroase și lichidul cefalorahidian. Carbenicilina în formă activă iar concentraţii mari se găsesc în bilă şi urină. Este produsă sub formă de sare disodică, deci dacă funcția rinichilor este afectată, pot apărea reținerea apei în organism și edem.

Utilizarea medicamentelor poate fi însoțită de apariția reacțiilor alergice, simptome de neurotoxicitate, nefrită interstițială acută, leucopenie, hipokaliemie, hipernatremie etc.

Ureidopeniciline (acilaminopeniciline)- piperacilina, mezlocilina, azlocilina sunt antibiotice cu spectru larg care suprimă microorganismele gram-pozitive și gram-negative. Aceste antibiotice sunt utilizate în principal pentru infecțiile cu gram-negative severe, în special pentru bolile cauzate de Pseudomonas aeruginosa (neapărat în combinație cu aminoglicozide), Klebsiella. Ureidopenicilinele pătrund bine în celule. Sunt puțin metabolizați în organism și sunt excretați de rinichi prin filtrare și secreție. Medicamentele sunt slab rezistente la B-lactamaze, de aceea se recomandă prescrierea lor cu inhibitori ai acestei enzime. Piperacilina este prescrisă pentru bolile inflamatorii cronice ale bronhiilor, inclusiv fibroza chistică și bronșită cronică. Medicamentele pot provoca leucopenie, trombocitopenie, neutropenie, eozinofilie, reacții alergice, disfuncție gastrointestinală, nefrită interstițială etc.

La numire peniciline semisintetice cu spectru larg: aminopeniciline (ampicilină, amoxicilină), carboxipeniciline (carbenicilină, ticarcilină), ureidopeniciline (piperacilină, mezlocilină, azlocilină), trebuie amintit că toate aceste antibiotice sunt distruse de B-lactamaze stafilococice și, prin urmare, producătoare de peniciline. sunt rezistente la actiunea lor.

Medicamente combinate cu inhibitori de B-lactamaze- acid clavulanic și sulbactam. Acidul clavulanic și sulbactam (sulfona acidului penicilanic) sunt clasificate ca B-lactamine, care au un efect antimicrobian foarte slab, dar în același timp suprimă activitatea B-lactamazelor stafilococilor și a altor microorganisme: Haemophilus influenzae, Escherichia coli, Klebsiella , unele bacteroide, gonococi, le -gionella; nu suprimați sau suprimați foarte slab B-lactamazele Pseudomonas aeruginosa, Enterobacteriaceae și Citrobacter. Preparatele care conțin acid clavulanic și sulbactam sunt destinate utilizării parenterale - augmentin (amoxicilină + clavulanat de potasiu), timentin (ticarcilină + clavulanat de potasiu), unasin (ampicilină + sulbactam). Sunt utilizate în tratamentul otitei medii, sinuzitei, infecțiilor tractului respirator inferior, pielii, țesuturilor moi, tractului urinar și a altor boli. Unazina este foarte eficientă pentru tratamentul peritonitei și meningitei cauzate de microorganisme care produc intens B-lactamaza. Analogii medicamentului unasin destinat administrării orale sunt sultamicilina și sulacilina.

Peniciline naturale și semisintetice(cu excepția carboxi- și ureidopenicilinelor) - antibiotice cu toxicitate scăzută. Cu toate acestea, benzilpenicilina și, într-o măsură mai mică, penicilinele semisintetice pot provoca reacții alergice și, prin urmare, utilizarea lor la copiii cu diateză și boli alergice este limitată. Introducere doze mari benzilpenicilina, ampicilină, amoxicilină, pot duce la creșterea excitabilității sistemului nervos central, convulsii, care este asociată cu antagonismul antibioticelor în raport cu transmițătorul inhibitor GABA din sistemul central. sistem nervos.

Preparate cu penicilină cu acțiune lungă trebuie injectat cu foarte mare atenție sub presiune ușoară printr-un ac de diametru mare. Dacă suspensia pătrunde într-un vas, poate provoca formarea de trombi. Penicilinele semisintetice utilizate pe cale orală provoacă iritații ale mucoasei gastrice, o senzație de greutate în abdomen, arsuri și greață, în special atunci când sunt administrate pe stomacul gol. Antibioticele cu spectru larg pot duce la disbiocenoză în intestine și pot provoca apariția unei infecții secundare cauzate de Pseudomonas aeruginosa, Klebsiella, drojdie etc. Pentru alte complicații cauzate de peniciline, vezi mai sus.

Cefalosporine- un grup de antibiotice naturale si semisintetice pe baza de acid 7-aminocefalosporanic.

În prezent, cea mai comună diviziune a cefalosporinelor este după generație.

Unele medicamente din acest grup pot fi utilizate pentru administrare orală: cefalosporine de prima generație - cefadroxil, cefalexină, cefradină; Generația a II-a - cefuroximă (zinnat), generația a III-a - cefspan (cefoximă), cefpodoximă (orelax), ceftibuten (cedex). Cefalosporinele orale sunt de obicei utilizate pentru boli moderate, deoarece sunt mai puțin active în comparație cu medicamentele pentru administrare parenterală.

Cefalosporinele au un spectru larg de acțiune.

Cefalosporinele de generația I inhibă activitatea cocilor, în special a stafilococilor și streptococilor (cu excepția enterococilor și a tulpinilor de stafilococi rezistente la meticilină), precum și a bacililor difteriei, bacililor antraxului, spirochetelor, Escherichia, Shigella, Salmonella, Moraxella, Yersilebsiella. , Bordetella, Protea și hemophilus influenzae. Cefalosporinele de a doua generație au același spectru de acțiune, dar creează mai mult concentratii mariîn sânge și pătrund în țesuturi mai bine decât medicamentele de prima generație. Au un efect mai activ asupra unor tulpini de bacterii gram-negative care sunt rezistente la prima generație de cefalosporine, inclusiv majoritatea tulpinilor de Escherichia coli, Klebsiella, Proteus, Haemophilus influenzae, Moraxella, agenți patogeni de tuse convulsivă și gonococi. În același timp, cefalosporinele de a doua generație nu afectează Pseudomonas aeruginosa, „tulpinile spitalicești” de bacterii gram-negative și au un efect puțin mai puțin inhibitor în comparație cu cefalosporinele din prima generație asupra stafilococilor și streptococilor. Cefalosporinele de generația a III-a se caracterizează printr-o lățime și mai mare a spectrului antimicrobian, o bună capacitate de penetrare și o activitate ridicată împotriva bacteriilor gram-negative, inclusiv a tulpinilor nosocomiale rezistente la alte antibiotice. Acestea afectează, pe lângă microbii menționați mai sus, pseudomonadele, morganella, zimăriturile, clostridiile (cu excepția CY. difficile) și bacteroidele. În același timp, se caracterizează printr-o activitate relativ scăzută împotriva stafilococilor, pneumococilor, meningococilor, gonococilor și streptococilor. Cefalosporinele de generația IV sunt mai active decât medicamentele de generația a III-a în suprimarea majorității bacteriilor gram-negative și gram-pozitive. Cefalosporinele de generația IV afectează unele microorganisme multirezistente care sunt rezistente la majoritatea antibioticelor: Cytobacter, Enterobacter, Acinetobacter.

Cefalosporinele de generația IV sunt rezistente la B-lactamaze și nu induc formarea lor. Dar ele nu afectează SY. difficile, bacteroides, enterococi, listeria, legionella și alte câteva microorganisme.

Sunt utilizate pentru tratarea bolilor severe, precum și la pacienții cu neutropenie și imunitate suprimată.

Cele mai mari concentrații de cefalosporine se găsesc în rinichi și țesutul muscular, concentrații mai mici se găsesc în plămâni, ficat, pleural și lichidele peritoneale. Toate cefalosporinele trec ușor prin placentă. Cefaloridina (Zeporin), cefotaxima (Claforan), moxalactamul (Latamoxef), ceftriaxona (Longacef), ceftizoxima (Epocelin) etc. patrund in lichidul cefalorahidian Majoritatea cefalosporinelor sunt excretate nemodificata de rinichi prin secretia activa de catre celulele tubulare globulare si partial. filtrare.

Cefalosporinele sunt utilizate în tratamentul bolilor cauzate de microorganisme rezistente la penicilină, uneori în prezența reacțiilor alergice la peniciline. Sunt prescrise pentru sepsis, boli ale sistemului respirator, ale tractului urinar, tractului gastro-intestinal, țesuturilor moi și oaselor. Pentru meningita la nou-născuții prematuri, a fost detectată activitate ridicată a cefotaximei, moxalactamului, ceftizoximei și ceftriaxonei.

Utilizarea cefalosporinelor poate fi însoțită de durere la locul injectării intramusculare; flebită după administrare intravenoasă; greață, vărsături, diaree atunci când luați medicamente pe cale orală. Cu utilizarea repetată, copiii cu sensibilitate ridicată la medicament pot prezenta erupții cutanate, febră și eozinofilie. Cefalosporinele nu sunt recomandate copiilor cu o reacție anafilactică la peniciline, dar utilizarea lor este acceptabilă în prezența altor manifestări de alergie - febră, erupții cutanate, etc. Reacții alergice încrucișate între cefalosporine și peniciline se observă în 5-10% din cazuri. Unele cefalosporine, în special cefaloridina și cefalotina, sunt nefrotoxice. Acest efect este asociat cu excreția lor lentă de către rinichi și cu acumularea de produse de peroxidare a lipidelor în ei. Nefrotoxicitatea antibioticului crește odată cu deficitul de vitamina E și seleniu. Medicamentele pot inhiba microflora tractului gastrointestinal și pot duce la disbiocenoză, infecții încrucișate cauzate de tulpinile de microbi din spital, candidoză și deficit de vitamina E în organism.

Aztreonii- sintetic de mare eficiență (antibiotic 3-lactamic din grupa monobactamelor. Folosit pentru tratamentul infecțiilor tractului respirator, meningitei, bolilor septice cauzate de gram-negative, inclusiv microorganismele multirezistente (pseudomonas, moraxella, klebsiella, hemophilus influenzae, E. coli, yersinia, serracia , enterobacter, meningococ, gonococ, salmonella, morganella). Aztreonam nu afectează bacteriile aerobe și anaerobe gram-pozitive.

Imipenem- (antibiotic 3-lactamic din grupa carbapenemilor cu un spectru de acțiune ultra-larg, incluzând majoritatea bacteriilor aerobe și anaerobe gram-pozitive și gram-negative, inclusiv microorganisme rezistente la peniciline, cefalosporine, aminoglicozide și alte antibiotice. Activitate bactericidă ridicată. a imipenemului se datorează pătrunderii ușoare prin pereții bacteriilor, cu un grad ridicat de afinitate pentru enzimele implicate în sinteza peretelui bacterian al microorganismelor.În prezent, din grupa menționată de antibiotice, imipenemul este utilizat în clinică în combinație cu cilastatină. (această combinație se numește tienam). Cilastatin inhibă peptidaza renală, inhibând astfel formarea metaboliților nefrotoxici ai imipenemului. Thienam are activitate antimicrobiană puternică, un spectru larg de acțiune. Sarea de sodiu a imipenem-cilastatin este produsă sub denumirea de Primaxin. Imipenem este stabil la (3-lactamaza, dar are un efect slab asupra microorganismelor situate în interiorul celulelor. Când se prescrie imipenem, pot apărea tromboflebite, diaree și, în cazuri rare, convulsii (în special cu afectarea funcției renale și boli ale sistemului nervos central) .

Meronem (meropenem) nu suferă biotransformare în rinichi și nu produce metaboliți nefrotoxici. Prin urmare, se utilizează fără cilastatină. Are mai puțin efect decât tienam asupra stafilococilor, dar este mai eficient împotriva enterobacteriilor gram-negative și a pseudomonadelor.

Meronem creează o concentrație bactericidă activă în lichidul cefalorahidian (LCR) și este utilizat cu succes pentru meningită fără teama de efecte nedorite. Acest lucru se compară favorabil cu tienam, care provoacă efecte neurotoxice și, prin urmare, este contraindicat pentru meningită.

Aztreonamul și carbapenemul practic nu sunt absorbite în tractul gastrointestinal și sunt administrate parenteral. Ele pătrund bine în majoritatea fluidelor și țesuturilor corporale și sunt excretate în principal prin urină într-o formă activă. S-a observat că medicamentele sunt foarte eficiente în tratarea pacienților cu infecții ale tractului urinar, sistemului osteoarticular, pielii, țesuturilor moi, infecții ginecologice și gonoree. Utilizarea aztreonamului este indicată în special în practica pediatrica ca alternativă la antibioticele aminoglicozide.

Fosfomicină (fosfonomicină)- un antibiotic bactericid cu spectru larg care perturbă formarea peretelui microbian prin suprimarea sintezei acidului UDP-acetilmuramic, adică mecanismul său de acțiune diferă de cel al penicilinelor și cefalosporinelor. Are un spectru larg de acțiune. Este capabil să suprime bacteriile gram-negative și gram-pozitive, dar nu afectează Klebsiella, Proteus indol-pozitiv.

Fosfomicina pătrunde bine în țesuturi, inclusiv în oase și în lichidul cefalorahidian; se găseşte în cantităţi suficiente în bilă. Antibioticul numit este excretat în principal prin rinichi. Este prescris în principal pentru infecții severe cauzate de microorganisme rezistente la alte antibiotice. Se combină bine cu penicilinele, cefalosporinele, iar atunci când este utilizat împreună cu antibiotice aminoglicozide, se observă nu numai o creștere a acțiunii antimicrobiene, ci și o scădere a nefrotoxicității acestora din urmă. Fosfomicina este eficientă în tratamentul meningitei, sepsisului, osteomielitei, infecțiilor urinare și ale căilor biliare. Pentru infecții bucale și infectii intestinale se prescrie enteral. Fosfomicina este un medicament cu toxicitate scăzută. Când îl folosesc, unii pacienți pot prezenta greață și diaree; până în prezent nu au fost identificate alte reacții adverse.

Antibiotice glicopeptidice. Vancomicina, teicoplanina sunt antibiotice care acționează asupra cocilor gram-pozitivi (inclusiv stafilococi rezistenți la meticilină, tulpini de stafilococi care formează B-lactamaze, streptococi, pneumococi rezistenți la penicilină, enterococi) și bacterii (corinebacterii etc.). Efectul lor asupra clostridiilor, în special difficile, este foarte important. Vancomicina afectează și actinomicetele.

Vancomicina pătrunde bine în toate țesuturile și fluidele corpului, cu excepția lichidului cefalorahidian. Este folosit pentru sever infecții cu stafilococ cauzate de tulpini rezistente la alte antibiotice. Principalele indicații ale vancomicinei sunt: ​​sepsis, infecții ale țesuturilor moi, osteomielita, endocardită, pneumonie, enterocolită necrozantă (provocată de clostridii toxigene). Vancomicina se administrează intravenos de 3-4 ori pe zi, la nou-născuți de 2 ori pe zi. În tratamentul meningitei stafilococice foarte severe, dată fiind pătrunderea relativ slabă a vancomicinei în lichidul cefalorahidian, se recomandă administrarea sa intratecală. Teicoplanina diferă de vancomicină prin eliminarea lentă; se administrează intravenos o dată pe zi. Pentru colita pseudomembranoasă și enterocolita stafilococică, vancomicina este prescrisă pe cale orală.

Cel mai complicatie comuna utilizarea masivă a vancomicinei - eliberarea histaminei din mastocite, ceea ce duce la hipotensiune arterială, apariția unei erupții cutanate roșii pe gât (sindromul gâtului roșu), cap și membre. Această complicație poate fi de obicei evitată dacă doza necesară de vancomicină este administrată în decurs de cel puțin o oră și antihistaminice pre-administrate. Tromboflebita și întărirea venelor sunt posibile în timpul perfuziei medicamentului. Vancomicina este un antibiotic nefrotoxic; utilizarea sa combinată cu aminoglicozide și alte medicamente nefrotoxice trebuie evitată. Vancomicina poate provoca convulsii atunci când este administrată intratecal.

Ristomicină (ristocetină)- un antibiotic care suprima microorganismele gram-pozitive. Stafilococii, streptococii, enterococii, pneumococii, bacilii gram-pozitivi care formează spori, precum și corinebacterii, listeria, bacteriile acido-resistente și unii anaerobi sunt sensibili la aceasta. Nu afectează bacteriile gram-negative și cocii. Ristomicina se administrează numai intravenos, nu este absorbită din tractul gastrointestinal. Antibioticul pătrunde bine în țesuturi; în special concentrații mari se găsesc în plămâni, rinichi și splină. Ristomicina este utilizată în principal pentru boli septice severe cauzate de stafilococi și enterococi în cazurile în care tratamentul anterior cu alte antibiotice a fost ineficient.

Când se utilizează ristomicina, se observă uneori trombocitopenie, leucopenie, neutropenie (până la agranulocitoză), iar uneori se observă eozinofilie. În primele zile de tratament, sunt posibile reacții de exacerbare (frisoane, erupții cutanate), reacții alergice sunt destul de des observate. Administrarea intravenoasă pe termen lung a ristomicinei este însoțită de întărirea pereților venelor și tromboflebită. Au fost descrise reacții oto- și nefrotoxice.

Polimixine- un grup de antibiotice bactericide polipeptidice care suprimă activitatea microorganismelor predominant gram-negative, inclusiv Shigella, Salmonella, tulpini enteropatogene de Escherichia coli, Yersinia, Vibrio cholerae, Enterobacter, Klebsiella. De mare importanță pentru pediatrie este capacitatea polimixinelor de a suprima activitatea Haemophilus influenzae și a majorității tulpinilor de Pseudomonas aeruginosa. Polimixinele acționează atât asupra microorganismelor în diviziune, cât și asupra microorganismelor latente. Dezavantajul polimixinelor este pătrunderea lor scăzută în celule și, prin urmare, eficiența scăzută în bolile cauzate de agenți patogeni intracelulari (bruceloză, febră tifoidă). Polimixinele se caracterizează printr-o pătrundere slabă prin barierele tisulare. Când sunt luate pe cale orală, practic nu sunt absorbite. Polimixinele B și E se folosesc intramuscular, intravenos, pentru meningită se administrează endolombar, pentru infecții gastrointestinale se prescriu oral. Polimixina M este utilizată numai intern și local. Polimixinele orale sunt prescrise pentru dizenterie, holeră, colienterită, enterocolită, gastroenterocolită, salmoneloză și alte infecții intestinale.

Când polimixinele sunt prescrise pe cale orală, precum și atunci când sunt aplicate local, reacțiile adverse sunt rareori observate. Când sunt administrate parenteral, pot provoca efecte nefro- și neurotoxice (neuropatii periferice, tulburări de vedere și vorbire, slăbiciune musculară). Aceste complicații sunt cele mai frecvente la persoanele cu funcție excretorie renală afectată. Febră, eozinofilie și urticarie sunt uneori observate atunci când se utilizează polimixine. La copii, administrarea parenterală a polimixinelor este permisă numai din motive de sănătate, în cazul proceselor infecțioase cauzate de microflora gram-negativă, care sunt rezistente la acțiunea altor medicamente antimicrobiene, mai puțin toxice.

Gramicidin (Gramicidin C) activ în principal împotriva microflorei gram-pozitive, inclusiv a streptococilor, stafilococilor, pneumococilor și a altor microorganisme. Gramicidina este utilizată numai local sub formă de pastă, soluții și tablete bucale. Soluțiile de gramicidină sunt utilizate pentru tratarea pielii și a membranelor mucoase, pentru spălarea, irigarea bandajelor în tratamentul escarelor, rănilor purulente, furunculelor etc. Tabletele de gramicidină sunt destinate resorbției atunci când procese infecțioaseîn cavitatea bucală și faringe (dureri în gât, faringită, stomatită etc.). Comprimatele de gramicidină nu trebuie înghițite: dacă intră în sânge, pot provoca hemoliza eritromiților.

Macrolide. Există trei generații de macrolide. Generația I - eritromicină, oleandomicină. Generația a II-a - spiramicină (Rovamycin), roxitromicină (Rulid), josamicina (Vilprafen), claritromicină (Cladid), midecamicină (Macropen). Generația a III-a - azitromicină (sumamed).

Macrolidele sunt antibiotice cu spectru larg. Au efect asupra microorganismelor care sunt foarte sensibile la acestea. efect bactericid: stafilococi, streptococi, pneumococi, corinebacterii, bordetella, moraxella, chlamydia și micoplasme. Ele afectează alte microorganisme - Neisseria, Legionella, Haemophilus influenzae, Brucella, Treponema, Clostridia și Rickettsia - bacteriostatic. Macrolidele din generațiile II și III au un spectru mai larg de acțiune. Astfel, josamicina și claritromicina suprimă Helicobacter pylori (și sunt utilizate în tratamentul ulcerului gastric), spiramicina afectează toxoplasma. Preparatele din generațiile II și III inhibă și bacteriile gram-negative: Campylobacter, Listeria, Gardnerella și unele micobacterii.

Toate macrolidele pot fi administrate pe cale orală, unele medicamente (fosfat de eritromicină, spiramicină) pot fi administrate intravenos.

Macrolidele pătrund bine în adenoizi, amigdale, țesuturi și fluide ale urechii medii și interne, țesut pulmonar, bronhii, secreție bronșică iar sputa, pielea, lichidele pleurale, peritoneale si sinoviale, se gasesc in concentratii mari in neutrifile si macrofage alveolare. Macrolidele pătrund slab în lichidul cefalorahidian și în sistemul nervos central. De mare importanță este capacitatea lor de a pătrunde în celule, de a se acumula în ele și de a suprima infecția intracelulară.

Medicamentele sunt eliminate în principal de ficat și creează concentrații mari în bilă.

Noile macrolide se deosebesc de cele vechi prin o mai mare stabilitate într-un mediu acid și o mai bună bio-absorbție din tractul gastrointestinal, indiferent de aportul alimentar, și acțiune prelungită.

Macrolidele sunt prescrise în principal pentru formele ușoare boli acute cauzate de microorganisme sensibile la acestea. Principalele indicații pentru utilizarea macrolidelor sunt amigdalita, pneumonia (inclusiv cele cauzate de Legionella), bronșita, difteria, tusea convulsivă, otita purulentă, bolile ficatului și ale căilor biliare, pneumopatia și conjunctivita cauzate de chlamydia. Sunt foarte eficiente împotriva pneumoniei cu chlamydia la nou-născuți. Macrolidele sunt folosite și pentru boli ale tractului urinar, dar pentru a obține un efect terapeutic bun, mai ales atunci când se folosesc macrolide „vechi”, urina trebuie alcalinizată, deoarece acestea sunt inactive într-un mediu acid. Sunt prescrise când sifilisul primarși gonoree.

Sinergismul se observă atunci când macrolidele sunt utilizate împreună cu medicamente sulfonamide și antibiotice tetracicline. Preparatele combinate care conțin oleandromicină și tetracicline sunt comercializate sub denumirile de oleetrin, tetraolean și sigmamicină. Macrolidele nu pot fi combinate cu cloramfenicol, peniciline sau cefalosporine.

Macrolidele sunt antibiotice cu toxicitate scăzută, dar irită membrana mucoasă a tractului gastrointestinal și pot provoca greață, vărsături și diaree. Injecțiile intramusculare sunt dureroase, iar flebita se poate dezvolta cu injecția intravenoasă. Uneori, atunci când sunt folosite, se dezvoltă colestaza. Eritromicina și alte macrolide inhibă sistemul monooxigenazei din ficat, drept urmare biotransformarea unui număr de medicamente, în special teofilinei, este întreruptă, crescând astfel concentrația acestuia în sânge și toxicitatea. De asemenea, inhibă biotransformarea bromocriptinei, dihidroergotaminei (incluse într-o serie de medicamente antihipertensive), carbamazepinei, cimetidinei etc.

Microlidele nu pot fi prescrise împreună cu noi antihistaminice - terfenadină și astemizol din cauza pericolului acțiunii lor hepatoxice și a pericolului de aritmie cardiacă.

Lincosamide: lincomicină și clindamicină. Aceste antibiotice suprimă predominant microorganisme gram-pozitive, inclusiv stafilococi, streptococi, pneumococi, precum și micoplasme, diverse bacterii, fusobacterii, coci anaerobi și unele tulpini de Haemophilus influenzae. Clindamicina, în plus, are un efect, deși slab, asupra toxoplasmei, agenților cauzatori ai malariei și a gangrenei gazoase. Majoritatea bacteriilor gram-negative sunt rezistente la lincosamide.

Lincosamidele sunt bine absorbite din tractul gastrointestinal, indiferent de aportul alimentar, pătrund în aproape toate fluidele și țesuturile, inclusiv oasele, dar penetrează slab în sistemul nervos central și lichidul cefalorahidian. Pentru nou-născuți, medicamentele se administrează de 2 ori pe zi, pentru copiii mai mari - de 3-4 ori pe zi.

Clindamicina diferă de lincomicină prin activitatea sa mai mare împotriva anumitor tipuri de microorganisme și o absorbție mai bună din tractul gastrointestinal, dar în același timp provoacă mai des reacții nedorite.

Lincosamidele sunt utilizate în tratamentul infecțiilor cauzate de microorganisme gram-pozitive rezistente la alte antibiotice, în special în cazurile de alergie la medicamentele peniciline și cefalosporine. Sunt prescrise pentru boli ginecologice infecțioase și infecții gastrointestinale. Datorită pătrunderii bune în țesutul osos, lincosamidele sunt medicamentele de elecție în tratamentul osteomielitei. Fără indicații speciale, acestea nu trebuie prescrise copiilor atunci când alte antibiotice, mai puțin toxice sunt eficiente.

Când folosesc lincosamide, copiii pot prezenta greață și diaree. Uneori se dezvoltă colita pseudomembranoasă - o complicație severă cauzată de disbiocenoză și reproducere în intestinul Cy. difficile, care secretă o toxină. Aceste antibiotice pot provoca disfuncție hepatică, icter, leuconeutropenie și trombocitopenie. Reacțiile alergice, în principal sub formă de erupții cutanate, sunt destul de rare. Prin administrare intravenoasă rapidă, lincosamidele pot provoca bloc neuromuscular cu depresie respiratorie și colaps.

Fuzidin. Cea mai mare valoare Fusidina are activitate împotriva stafilococilor, inclusiv a celor rezistenți la alte antibiotice. De asemenea, acționează asupra altor coci gram-pozitivi și gram-negativi (gonococi, meningococi). Fuzidin este oarecum mai puțin activ împotriva corynebacteriilor, listeriei și clostridiilor. Antibioticul nu este activ împotriva tuturor bacteriilor și protozoarelor gram-negative.

Fusidina este bine absorbită din tractul gastrointestinal și pătrunde în toate țesuturile și lichidele, cu excepția lichidului cefalorahidian. Antibioticul pătrunde foarte bine în sursa de inflamație, ficat, rinichi, piele, cartilaj, oase și secreții bronșice. Preparatele cu fusidină sunt prescrise oral, intravenos și, de asemenea, local sub formă de unguent.

Fusidina este indicată în special pentru bolile cauzate de tulpinile de stafilococi rezistente la penicilină. Medicamentul este foarte eficient pentru osteomielita, boli ale sistemului respirator, ficat, tract biliar și piele. În ultimii ani, a fost utilizat în tratamentul pacienților cu nocardioză și colită cauzate de clostridii (cu excepția CY. difficile). Fusidina se excretă în principal în bilă și poate fi utilizată la pacienții cu insuficiență a funcției excretorii renale.

O creștere pronunțată a activității antimicrobiene se observă atunci când fusidina este combinată cu alte antibiotice; combinația cu tetracicline, rifampicină și aminoglicozide este deosebit de eficientă.

Fuzidin este un antibiotic cu toxicitate scăzută, dar poate provoca tulburări dispeptice care dispar după întreruperea medicamentului. Când un antibiotic este administrat intramuscular, se observă necroză tisulară (!), iar atunci când este administrat intravenos poate apărea tromboflebită.

Antibiotice aminoglicozide. Există patru generații de aminoglicozide. Antibioticele de prima generație includ streptomicina, monomicină, neomicină, kanamicina; generația a II-a - gentamicina (garamicină); generația a III-a - tobramicină, sisomicina, amikacină, netilmicină; generația IV - isepamicină.

Antibioticele aminoglicozide sunt bactericide, au un spectru larg de acțiune și inhibă microorganismele gram-pozitive și în special gram-negative. Aminoglicozidele din generațiile II, III și IV sunt capabile să suprime Pseudomonas aeruginosa. De importanță practică principală este capacitatea medicamentelor de a inhiba activitatea patogenelor Escherichia coli, Haemophilus influenzae, Klebsiella, gonococi, Salmonella, Shigella și stafilococi. În plus, streptomicina și kanamicina sunt utilizate ca medicamente anti-tuberculoză, monomicină pentru a acționa asupra amibei dizenterice, leishmania, trichomonas, gentamicina - asupra agentului cauzal al tularemiei.

Toate antibioticele aminoglicozide sunt slab absorbite din tractul gastrointestinal și din lumenul bronșic. Pentru a obține un efect de resorbție, se administrează intramuscular sau intravenos. După o singură injecție intramusculară concentrare eficientă medicamentul din plasma sanguină rămâne la nou-născuți și copii mici timp de 12 ore sau mai mult, la copiii mai mari și adulți timp de 8 ore.Mercamentul pătrunde satisfăcător în țesuturi și fluide corporale, cu excepția lichidului cefalorahidian, ele pătrund slab în celule. . La tratarea meningitei cauzate de bacterii gram-negative, antibioticele aminoglicozide sunt de preferință administrate pe cale endolombară. În prezența unui proces inflamator sever în plămâni, organe abdominale, pelvis, osteomielita și sepsis, este indicată administrarea endolimfatică de medicamente, care asigură o concentrație suficientă a antibioticului în organe fără a provoca acumularea acestuia în rinichi. La bronșită purulentă se administrează sub formă de aerosol sau prin instalarea unei soluţii direct în lumenul bronhiilor. Antibioticele din acest grup trec bine prin placentă și sunt excretate în lapte (la un sugar, aminoglicozidele practic nu sunt absorbite din tractul gastrointestinal), dar există un risc mare de disbacterioză.

La administrarea repetată se produce o acumulare de aminoglicozide în flacon, în timpul urechea internăși alte câteva organe.

Drogurile nu sunt. suferă biotransformare și sunt excretate de rinichi într-o formă activă. Eliminarea antibioticelor aminoglicozide este încetinită la nou-născuți, în special la prematuri, precum și la pacienții cu insuficiență a funcției excretorii renale.

Antibioticele aminoglicozide sunt utilizate pentru boli infecțioase complicate ale tractului respirator și urinar, pentru septicemie, endocardită și mai rar pentru infecții ale tractului gastrointestinal, pentru prevenire și tratament. complicatii infectioase la pacientii operati.

Antibioticele aminoglicozide administrate parenteral sunt toxice. Ele pot provoca efecte ototoxice, nefrotoxice, pot perturba transmiterea neuromusculară a impulsurilor și procesele de absorbție activă din tractul gastrointestinal.

Efectul ototoxic al antibioticelor este o consecință a ireversibile modificări degenerative celulele capilare ale organului lui Corti (urechea internă). Riscul ca acest efect să apară este cel mai mare la nou-născuți, în special la prematuri, precum și la traumatisme la naștere, hipoxie în timpul nașterii, meningită, afectarea funcției excretorii renale. Un efect ototoxic se poate dezvolta atunci când antibioticele ajung la făt prin placentă; atunci când este combinat cu alte medicamente ototoxice (furosemid, acid etacrinic, ristomicina, antibiotice glicopeptidice).

Efectul nefrotoxic al antibioticelor aminoglicozide este asociat cu disfuncția multor enzime în celule epiteliale tubuli renali, distrugerea lizozomilor. Clinic, aceasta se manifestă prin creșterea volumului urinei, scăderea concentrației sale și proteinurie, adică apariția insuficienței renale non-oligurice.

Antibioticele din acest grup nu pot fi combinate cu alte medicamente oto- și nefrotoxice. La copiii mici, în special copiii subnutriți și slăbiți, antibioticele aminoglicozide pot inhiba transmiterea neuromusculară datorită scăderii sensibilității receptorilor H-colinergici ai mușchilor scheletici la acetilcolină și suprimării eliberării transmițătorului; Ca urmare, funcția mușchilor respiratori poate fi afectată. Pentru a elimina această complicație, preparatele de calciu sunt prescrise împreună cu proserina după administrarea preliminară de atropină. Acumulându-se în peretele intestinal, aminoglicozidele perturbă procesul de absorbție activă a aminoacizilor, vitaminelor și zaharurilor. Acest lucru poate duce la malabsorbție, care agravează starea copilului. Când sunt prescrise antibiotice aminoglicozide, concentrația de magneziu și calciu în plasma sanguină scade.

Datorită toxicității lor ridicate, antibioticele aminoglicozide trebuie prescrise numai pentru infecții severe, în cure scurte (nu mai mult de 5-7 zile).

Levomicetina- un antibiotic bacteriostatic, dar are efect bactericid asupra Haemophilus influenzae tip “B”, a unor tulpini de meningococi si pneumococi. Inhibă diviziunea multor bacterii gram-negative: Salmonella, Shigella, Escherichia coli, Brucella, patogen pentru tuse convulsivă; coci aerobi gram-pozitivi: streptococi piogeni și streptococi de grup B; majoritatea microorganismelor anaerobe (clostridii, bacteroides); Vibrio cholerae, rickettsia, chlamydia, micoplasma.

Micobacteriile sunt rezistente la cloramfenicol, CI. difficile, Cytobacter, Enterobacter, Acinetobacter, Proteus, Pseudomonas aeruginosa, Staphylococcus, Enterococcus, Corynebacterium, Serration, protozoare și ciuperci.

Baza de levomicetina este bine absorbită din tractul gastrointestinal, creând rapid concentrații active în plasma sanguină. Antibioticul pătrunde bine din plasma sanguină în toate țesuturile și lichidele, inclusiv în lichidul cefalorahidian.

Din păcate, cloramfenicolul în sine are un gust amar și poate provoca vărsături la copii, așa că vârstă mai tânără ei preferă să prescrie esteri de cloramfenicol – stearat sau palmitat. La copiii din primele luni de viață, absorbția cloramfenicolului prescris sub formă de esteri are loc lent datorită activității scăzute a lipazelor care hidrolizează legăturile esterice și eliberează baza de cloramfenicol, care este capabilă de absorbție. De asemenea, succinatul de cloramfenicol administrat intravenos suferă hidroliză (în ficat sau rinichi) cu eliberarea bazei active de cloramfenicol. Esterul nehidrolizat este excretat prin rinichi, la nou-născuți aproximativ 80% din doza administrată, la adulți 30%. Activitatea hidrolazelor la copii este scăzută și prezintă diferențe individuale, prin urmare, de la aceeași doză de cloramfenicol, pot apărea concentrații inegale în plasma sanguină și lichidul cefalorahidian, în special în vârstă fragedă. Este necesar să se controleze concentrația de cloramfenicol în sângele copilului, deoarece fără aceasta este posibil să nu obțineți un efect terapeutic sau să provocați intoxicație. Conținutul de cloramfenicol liber (activ) din plasma sanguină și lichidul cefalorahidian după administrarea intravenoasă este de obicei mai mic decât după administrarea orală.

Levomicetina este deosebit de importantă în tratamentul meningitei cauzate de Haemophilus influenzae, meningococi și pneumococi, asupra cărora are efect bactericid. Pentru a trata aceste meningite, cloramfenicolul este adesea combinat cu antibiotice B-lactamice (în special ampicilină sau amoxicilină). Pentru meningita cauzată de alți agenți patogeni, utilizare în comun cloramfenicolul cu peniciline este inadecvat, deoarece în astfel de cazuri sunt antagonişti. Levomicetina este utilizată cu succes în tratament febră tifoidă, febră paratifoidă, dizenterie, bruceloză, tularemie, tuse convulsivă, infecții oculare (inclusiv trahom), ureche medie, piele și multe alte boli.

Levomicetina este neutralizată în ficat și excretată prin rinichi. În cazul bolilor hepatice, din cauza perturbării biotransformării normale a cloramfenicolului, poate apărea intoxicația cu acesta. La copiii din primele luni de viață, neutralizarea acestui antibiotic are loc lent și, prin urmare, există un risc mare de acumulare de cloramfenicol liber în organism, ceea ce duce la o serie de reacții nedorite. În plus, levomicetina inhibă funcția hepatică și inhibă biotransformarea teofilinei, fenobarbitalului, difeninei, benzodiazepinelor și a unui număr de alte medicamente, crescând concentrația acestora în plasma sanguină. Administrarea concomitentă de fenobarbital stimulează neutralizarea cloramfenicolului în ficat și îi reduce eficacitatea.

Levomicetina este un antibiotic toxic. Odată cu o supradoză de cloramfenicol la nou-născuți, în special la prematuri și la copii în primele 2-3 luni de viață, poate apărea „colapsul gri”: vărsături, diaree, insuficiență respiratorie, cianoză, colaps cardiovascular, stop cardiac și respirator. Colapsul este o consecință a activității cardiace afectate din cauza inhibării fosforilării oxidative în mitocondrii.Fără ajutor, rata mortalității nou-născuților din „colapsul gri” este foarte mare (40% sau mai mult).

Cea mai frecventă complicație la prescrierea cloramfenicolului este o tulburare a hematopoiezei. Pot exista tulburări reversibile dependente de doză, cum ar fi anemie hipocromă(datorită utilizării afectate a fierului și sintezei hemului), trombocitopenie și leucopenie. După întreruperea tratamentului cu cloramfenicol, imaginea sanguină este restabilită, dar încet. Modificări ireversibile, independente de doză, ale hematopoiezei sub formă de anemie aplastică apar cu o frecvență de 1 la 20.000-1 la 40.000 de persoane care iau cloramfenicol și se dezvoltă de obicei la 2-3 săptămâni (dar poate fi 2-4 luni) după utilizarea antibioticului. Ele nu depind de doza de antibiotic și de durata tratamentului, dar sunt asociate cu caracteristici genetice biotransformarea cloramfenicolului. În plus, cloramfenicolul inhibă funcția ficatului, a cortexului suprarenal, a pancreasului și poate provoca nevrite și malnutriție. Reacțiile alergice la utilizarea cloramfenicolului sunt rare. Complicațiile biologice se pot manifesta sub formă de suprainfecții cauzate de microorganisme rezistente la antibiotice, disbiocenoză etc. Pentru copiii sub 3 ani, cloramfenicolul se prescrie doar pentru indicații speciale și numai în cazuri foarte severe.

Un antibiotic este o substanță anti-vie - un medicament care este utilizat pentru tratarea bolilor cauzate de agenți vii, de obicei diferite bacterii patogene.

Antibioticele sunt împărțite în mai multe tipuri și grupuri dintr-o varietate de motive. Clasificarea antibioticelor face posibilă determinarea cât mai eficientă a domeniului de aplicare a fiecărui tip de medicament.

1. În funcție de origine.

  • Natural (natural).
  • Semi-sintetic - în stadiul inițial de producție, substanța este obținută din materii prime naturale, iar apoi medicamentul continuă să fie sintetizat artificial.
  • Sintetic.

Strict vorbind, numai medicamentele obținute din materii prime naturale sunt antibiotice. Toate celelalte medicamente sunt numite „medicamente antibacteriene”. ÎN lumea modernă Termenul „antibiotic” înseamnă toate tipurile de medicamente care pot lupta împotriva agenților patogeni vii.

Din ce sunt făcute antibioticele naturale?

  • din mucegaiuri;
  • din actinomicete;
  • din bacterii;
  • din plante (fitoncide);
  • din țesuturi de pești și animale.

2. În funcție de impact.

  • Antibacterian.
  • Antitumoral.
  • Antifungic.

3. În funcție de spectrul de impact asupra unui anumit număr de microorganisme diferite.

  • Antibiotice cu spectru restrâns de acțiune.
    Aceste medicamente sunt de preferat pentru tratament, deoarece acționează în mod specific asupra unui anumit tip (sau grup) de microorganisme și nu suprimă microflora sănătoasă a corpului pacientului.
  • Antibiotice cu un spectru larg de efecte.

4. După natura efectului asupra celulei bacteriene.

  • Medicamente bactericide - distrug agenții patogeni.
  • Bacteriostatice – stopează creșterea și reproducerea celulelor. Ulterior, sistemul imunitar al organismului trebuie să facă față în mod independent bacteriilor rămase în interior.

5. După structura chimică.
Pentru cei care studiază antibioticele, clasificarea după structura chimică este decisivă, deoarece structura medicamentului determină rolul acestuia în tratamentul diferitelor boli.

1. Medicamente beta-lactamice

1. Penicilina este o substanță produsă de colonii de ciuperci de mucegai din specia Penicillium. Derivații naturali și artificiali de penicilină au efect bactericid. Substanța distruge pereții celulari ai bacteriilor, ceea ce duce la moartea acestora.

Bacteriile patogene se adaptează la medicamente și devin rezistente la acestea. Noua generație de peniciline este suplimentată cu tazobactam, sulbactam și acid clavulanic, care protejează medicamentul de distrugerea în interiorul celulelor bacteriene.

Din păcate, penicilinele sunt adesea percepute de organism ca un alergen.

Grupe de antibiotice peniciline:

  • Penicilinele de origine naturala nu sunt protejate de penicilinaza, o enzima care este produsa de bacteriile modificate si care distruge antibioticul.
  • Semisintetice – rezistente la enzimele bacteriene:
    penicilina G biosintetică - benzilpenicilină;
    aminopenicilină (amoxicilină, ampicilină, becampicilină);
    penicilină semisintetică (preparate din meticilină, oxacilină, cloxacilină, dicloxacilină, flucloxacilină).

2. Cefalosporină.

Folosit în tratamentul bolilor cauzate de bacterii rezistente la penicilină.

Astăzi sunt cunoscute 4 generații de cefalosporine.

  1. Cefalexină, cefadroxil, ceporină.
  2. Cefamezin, cefuroxima (Axetil), cefazolin, cefaclor.
  3. Cefotaximă, ceftriaxonă, ceftizadimă, ceftibuten, cefoperazonă.
  4. Cefpirom, cefepime.

Cefalosporinele provoacă, de asemenea, reacții alergice în organism.

Cefalosporinele sunt utilizate în timpul intervențiilor chirurgicale pentru prevenirea complicațiilor, în tratamentul bolilor ORL, gonoreei și pielonefritei.

2. Macrolide
Au un efect bacteriostatic - previn creșterea și divizarea bacteriilor. Macrolidele acționează direct pe locul inflamației.
Printre antibiotice moderne macrolidele sunt considerate cele mai puțin toxice și provoacă un minim de reacții alergice.

Macrolidele se acumulează în organism și sunt utilizate în cure scurte de 1-3 zile. Sunt utilizate în tratamentul inflamației organelor interne ORL, plămânilor și bronhiilor și infecțiilor organelor pelvine.

Eritromicină, roxitromicină, claritromicină, azitromicină, azalide și cetolide.

3. Tetraciclină

Un grup de medicamente de origine naturală și artificială. Au efect bacteriostatic.

Tetraciclinele sunt utilizate în tratamentul infecțiilor severe: bruceloză, antrax, tularemie, infecții respiratorii și ale tractului urinar. Principalul dezavantaj al medicamentului este că bacteriile se adaptează foarte repede la el. Tetraciclina este cea mai eficientă atunci când este aplicată local sub formă de unguente.

  • Tetracicline naturale: tetraciclina, oxitetraciclina.
  • Tetracicline semi-sentitice: clortetrină, doxiciclină, metaciclină.

4. Aminoglicozide

Aminoglicozidele sunt medicamente bactericide foarte toxice active împotriva bacteriilor aerobe gram-negative.
Aminoglicozidele distrug rapid și eficient bacteriile patogene chiar și cu imunitatea slăbită. Pentru a lansa mecanismul de distrugere a bacteriilor, sunt necesare condiții aerobe, adică antibioticele din acest grup nu „funcționează” în țesuturile moarte și organele cu circulație sanguină slabă (cavități, abcese).

Aminoglicozidele sunt utilizate în tratamentul următoarelor afecțiuni: sepsis, peritonită, furunculoză, endocardită, pneumonie, afectare bacteriană a rinichilor, infecții ale tractului urinar, inflamație a urechii interne.

Medicamente aminoglicozide: streptomicina, kanamicina, amikacina, gentamicina, neomicina.

5. Levomicetina

Un medicament cu un mecanism de acțiune bacteriostatic împotriva agenților patogeni bacterieni. Folosit pentru a trata infecțiile intestinale grave.

Un efect secundar neplăcut al tratamentului cu cloramfenicol este afectarea măduvei osoase, care perturbă producția de celule sanguine.

6. Fluorochinolone

Preparate cu un spectru larg de acțiune și un puternic efect bactericid. Mecanismul de acțiune asupra bacteriilor este de a perturba sinteza ADN-ului, ceea ce duce la moartea acestora.

Fluorochinolonele sunt utilizate pentru tratamentul topic al ochilor și urechilor datorită efectelor secundare puternice. Medicamentele afectează articulațiile și oasele și sunt contraindicate în tratamentul copiilor și femeilor însărcinate.

Fluorochinolonele sunt utilizate împotriva următorilor agenți patogeni: gonococ, shigella, salmonella, holera, micoplasmă, chlamydia, Pseudomonas aeruginosa, legionella, meningococ, mycobacterium tuberculosis.

Medicamente: levofloxacin, gemifloxacin, sparfloxacin, moxifloxacin.

7. Glicopeptide

Antibiotic cu acțiune mixtă asupra bacteriilor. Are un efect bactericid împotriva majorității speciilor și un efect bacteriostatic împotriva streptococilor, enterococilor și stafilococilor.

Preparate glicopeptidice: teicoplanină (targocid), daptomicina, vancomicina (vankacină, diatracină).

8. Antibiotice antituberculoase
Medicamente: ftivazidă, metazidă, saluzidă, etionamidă, protionamidă, izoniazidă.

9. Antibiotice cu efect antifungic
Ele distrug structura membranei celulelor fungice, provocând moartea acestora.

10. Medicamente antilepră
Folosit pentru a trata lepra: solusulfonă, diucifonă, diafenilsulfonă.

11. Medicamente antitumorale – antracicline
Doxorubicină, rubomicină, carminomicină, aclarubicină.

12. Lincosamide
În ceea ce privește proprietățile lor medicinale, sunt foarte aproape de macrolide, deși din punct de vedere al compoziției lor chimice sunt un grup complet diferit de antibiotice.
Medicament: delacin S.

13. Antibioticele care sunt utilizate în practica medicală, dar nu aparțin niciunei dintre clasificările cunoscute.
Fosfomicină, fusidină, rifampicină.

Tabel de medicamente - antibiotice

Clasificarea antibioticelor pe grupe, tabelul distribuie unele tipuri de medicamente antibacteriene în funcție de structura chimică.

Grup de droguri Droguri Scopul aplicatiei Efecte secundare
Penicilină Penicilină.
Aminopenicilină: ampicilină, amoxicilină, becampicilină.
Semisintetice: meticilină, oxacilină, cloxacilină, dicloxacilină, flucloxacilină.
Antibiotic cu un spectru larg de efecte. Reactii alergice
Cefalosporină Generația 1: cefalexină, cefadroxil, ceporină.
2: Cefamezin, cefuroxima (Axetil), cefazolin, cefaclor.
3: Cefotaximă, ceftriaxonă, ceftizadimă, ceftibuten, cefoperazonă.
4: Cefpirom, cefepime.
Operații chirurgicale (pentru prevenirea complicațiilor), boli ORL, gonoree, pielonefrită. Reactii alergice
Macrolide Eritromicină, roxitromicină, claritromicină, azitromicină, azalide și cetolide. Organe ORL, plămâni, bronhii, infecții pelvine. Mai puțin toxic, nu provoacă reacții alergice
Tetraciclină tetraciclina, oxitetraciclina,
clortetrină, doxiciclină, metaciclină.
Bruceloză, antrax, tularemie, infecții ale căilor respiratorii și ale tractului urinar. Rapid creează dependență
Aminoglicozide Streptomicina, kanamicina, amikacina, gentamicina, neomicina. Tratamentul sepsisului, peritonitei, furunculozei, endocarditei, pneumoniei, leziunilor bacteriene ale rinichilor, infecțiilor tractului urinar, inflamației urechii interne. Toxicitate ridicată
Fluorochinolone Levofloxacină, gemifloxacină, sparfloxacină, moxifloxacină. Salmonella, gonococ, holera, chlamydia, micoplasma, Pseudomonas aeruginosa, meningococul, shigella, legionella, mycobacterium tuberculosis. A afecta SIstemul musculoscheletal: articulatii si oase. Contraindicat copiilor și femeilor însărcinate.
Levomicetina Levomicetina Infecții intestinale Leziuni ale măduvei osoase

Clasificarea principală a medicamentelor antibacteriene se realizează în funcție de structura lor chimică.

Antibioticele sunt un grup imens de medicamente bactericide, fiecare dintre acestea fiind caracterizat de propriul spectru de acțiune, indicații de utilizare și prezența anumitor consecințe.

Antibioticele sunt substanțe care pot inhiba creșterea microorganismelor sau le pot distruge. Conform definiției GOST, antibioticele includ substanțe de origine vegetală, animală sau microbiană. În prezent, această definiție este oarecum depășită, deoarece au fost create un număr mare de medicamente sintetice, dar antibioticele naturale au servit drept prototip pentru crearea lor.

Istoria medicamentelor antimicrobiene începe în 1928, când A. Fleming a descoperit pentru prima dată penicilină. Această substanță a fost descoperită și nu creată, deoarece a existat întotdeauna în natură. În natura vie, este produs de ciuperci microscopice din genul Penicillium, protejându-se de alte microorganisme.

În mai puțin de 100 de ani, au fost create peste o sută de medicamente antibacteriene diferite. Unele dintre ele sunt deja depășite și nu sunt folosite în tratament, iar altele sunt doar introduse în practica clinică.

Cum acționează antibioticele?

Vă recomandăm să citiți:

Toate medicamentele antibacteriene pot fi împărțite în două grupuri mari în funcție de efectul lor asupra microorganismelor:

  • bactericid– provoacă direct moartea microbilor;
  • bacteriostatic– previne proliferarea microorganismelor. Incapabile să crească și să se reproducă, bacteriile sunt distruse de sistemul imunitar al unei persoane bolnave.

Antibioticele își exercită efectele în multe feluri: unele dintre ele interferează cu sinteza acizilor nucleici microbieni; altele interferează cu sinteza pereților celulari bacterieni, altele perturbă sinteza proteinelor, iar altele blochează funcțiile enzimelor respiratorii.

Grupuri de antibiotice

În ciuda diversității acestui grup de medicamente, toate pot fi clasificate în mai multe tipuri principale. Această clasificare se bazează pe structura chimică - medicamentele din același grup au un similar formula chimica, deosebindu-se unele de altele prin prezența sau absența anumitor fragmente moleculare.

Clasificarea antibioticelor implică prezența grupelor:

  1. Derivați de penicilină. Aceasta include toate medicamentele create pe baza primului antibiotic. În acest grup, se disting următoarele subgrupe sau generații de medicamente penicilină:
  • Benzilpenicilina naturală, care este sintetizată de ciuperci, și medicamente semisintetice: meticilină, nafcilină.
  • Medicamente sintetice: carbpenicilina și ticarcilina, care au un spectru de acțiune mai larg.
  • Mecillam și azlocilină, care au un spectru de acțiune și mai larg.
  1. Cefalosporine- Cele mai apropiate rude ale penicilinelor. Primul antibiotic din acest grup, cefazolina C, este produs de ciuperci din genul Cephalosporium. Majoritatea medicamentelor din acest grup au un efect bactericid, adică ucid microorganismele. Există mai multe generații de cefalosporine:
  • Generația I: cefazolină, cefalexină, cefradină etc.
  • Generația a II-a: cefsulodin, cefamandol, cefuroximă.
  • Generația a III-a: cefotaximă, ceftazidimă, cefodizimă.
  • Generația a IV-a: cefpirom.
  • Generația V: ceftolozan, ceftopibrol.

Diferențele dintre diferitele grupuri sunt în principal în eficacitatea lor - generațiile ulterioare au un spectru de acțiune mai mare și sunt mai eficiente. Cefalosporinele de generația I și a II-a sunt acum utilizate extrem de rar în practica clinică, majoritatea nici măcar nu sunt produse.

  1. – medicamente cu o structură chimică complexă care au un efect bacteriostatic asupra unei game largi de microbi. Reprezentanți: azitromicină, rovamicină, josamicina, leucomicină și o serie de altele. Macrolidele sunt considerate unul dintre cele mai sigure medicamente antibacteriene - pot fi folosite chiar și de femeile însărcinate. Azalidele și cetolidele sunt soiuri de macorlide care au diferențe în structura moleculelor active.

Un alt avantaj al acestui grup de medicamente este că sunt capabile să pătrundă în celule corpul uman, ceea ce le face eficiente in tratamentul infectiilor intracelulare: , .

  1. Aminoglicozide. Reprezentanți: gentamicină, amikacină, kanamicina. Eficient împotriva un numar mare microorganisme aerobe gram-negative. Aceste medicamente sunt considerate cele mai toxice și pot duce la complicații destul de grave. Folosit pentru tratarea infecțiilor tractului genito-urinar.
  2. Tetracicline. Acestea sunt în principal medicamente semisintetice și sintetice, care includ: tetraciclină, doxiciclină, minociclină. Eficient împotriva multor bacterii. Dezavantajul acestor medicamente este rezistența încrucișată, adică microorganismele care au dezvoltat rezistență la un medicament vor fi insensibile la altele din acest grup.
  3. Fluorochinolone. Acestea sunt medicamente complet sintetice care nu au omologul lor natural. Toate medicamentele din acest grup sunt împărțite în prima generație (pefloxacină, ciprofloxacină, norfloxacină) și a doua generație (levofloxacină, moxifloxacină). Ele sunt cel mai adesea utilizate pentru a trata infecțiile organelor ORL (,) și ale tractului respirator (,).
  4. Lincosamidele. Acest grup include antibioticul natural lincomicina și derivatul său clindamicină. Au atât efecte bacteriostatice, cât și bactericide, efectul depinde de concentrație.
  5. Carbapenemi. Acestea sunt unul dintre cele mai moderne antibiotice asupra cărora acționează un numar mare de microorganisme. Medicamentele din acest grup aparțin antibioticelor de rezervă, adică sunt utilizate cel mai mult cazuri dificile când alte medicamente sunt ineficiente. Reprezentanți: imipenem, meropenem, ertapenem.
  6. Polimixine. Acestea sunt medicamente foarte specializate utilizate pentru a trata infecțiile cauzate de. Polimixinele includ polimixinele M și B. Dezavantajul acestor medicamente este efectul lor toxic asupra sistemului nervos și rinichilor.
  7. Medicamente antituberculoase. Acesta este un grup separat de medicamente care au un efect pronunțat asupra. Acestea includ rifampicina, izoniazida și PAS. Pentru tratarea tuberculozei se folosesc și alte antibiotice, dar numai dacă s-a dezvoltat rezistență la medicamentele menționate.
  8. Agenți antifungici. Acest grup include medicamente utilizate pentru tratarea micozelor - infecții fungice: amfotirecină B, nistatina, fluconazol.

Metode de utilizare a antibioticelor

Medicamentele antibacteriene sunt disponibile în diferite forme: tablete, pulbere din care se prepară o soluție injectabilă, unguente, picături, spray, sirop, supozitoare. Principalele utilizări ale antibioticelor:

  1. Oral- administrare orală. Puteți lua medicamentul sub formă de tabletă, capsule, sirop sau pulbere. Frecvența administrării depinde de tipul de antibiotic, de exemplu, azitromicina se administrează o dată pe zi, iar tetraciclina se administrează de 4 ori pe zi. Pentru fiecare tip de antibiotic există recomandări care indică când trebuie luat - înainte, în timpul sau după mese. Eficacitatea tratamentului și severitatea efectelor secundare depind de aceasta. Antibioticele sunt uneori prescrise copiilor mici sub formă de sirop - este mai ușor pentru copii să bea lichidul decât să înghită o tabletă sau o capsulă. În plus, siropul poate fi îndulcit pentru a elimina gustul neplăcut sau amar al medicamentului în sine.
  2. Injectabil– sub formă de injecții intramusculare sau intravenoase. Cu această metodă, medicamentul ajunge mai repede la locul infecției și este mai activ. Dezavantajul acestei metode de administrare este că injecția este dureroasă. Injecțiile sunt utilizate pentru boli moderate și severe.

Important:Doar o asistentă ar trebui să facă injecții într-o clinică sau spital! Nu este strict recomandat să injectați antibiotice acasă.

  1. Local– aplicarea de unguente sau creme direct pe locul infecției. Această metodă de administrare a medicamentelor este utilizată în principal pentru infecții ale pielii - erizipel, precum și în oftalmologie - pentru bolile infecțioase ale ochiului, de exemplu, unguent cu tetraciclină pentru conjunctivită.

Calea de administrare este determinată numai de medic. În acest caz, sunt luați în considerare mulți factori: absorbția medicamentului în tractul gastrointestinal, starea sistemului digestiv în ansamblu (în unele boli, rata de absorbție scade și eficacitatea tratamentului scade). Unele medicamente pot fi administrate doar într-un singur mod.

Când injectați, trebuie să știți cum să dizolvați pulberea. De exemplu, Abactal poate fi diluat doar cu glucoză, deoarece atunci când este utilizată clorură de sodiu, aceasta este distrusă, ceea ce înseamnă că tratamentul va fi ineficient.

Sensibilitate la antibiotice

Orice organism mai devreme sau mai târziu se obișnuiește cu cele mai dure condiții. Această afirmație este adevărată și în legătură cu microorganismele - ca răspuns la expunerea prelungită la antibiotice, microbii dezvoltă rezistență la acestea. Conceptul de sensibilitate la antibiotice a fost introdus în practica medicală - eficacitatea cu care un anumit medicament afectează agentul patogen.

Orice prescripție de antibiotice ar trebui să se bazeze pe cunoașterea sensibilității agentului patogen. În mod ideal, înainte de a prescrie un medicament, medicul ar trebui să efectueze un test de sensibilitate și să prescrie cel mai eficient medicament. Dar momentul unei astfel de analize este cel mai bun scenariu– câteva zile, iar în acest timp infecția poate duce la cel mai tragic rezultat.

Prin urmare, în caz de infecție cu un agent patogen necunoscut, medicii prescriu medicamente în mod empiric - ținând cont de cel mai probabil agent patogen, cu cunoașterea situației epidemiologice într-o anumită regiune și institutie medicala. În acest scop, se folosesc antibiotice cu spectru larg.

După efectuarea unui test de sensibilitate, medicul are posibilitatea de a schimba medicamentul cu unul mai eficient. Medicamentul poate fi înlocuit dacă tratamentul nu are efect timp de 3-5 zile.

Prescrierea etiotropă (țintită) a antibioticelor este mai eficientă. În același timp, devine clar ce a cauzat boala - cu ajutorul cercetare bacteriologică se determină tipul de agent patogen. Apoi, medicul selectează un anumit medicament la care microbul nu are rezistență (rezistență).

Antibioticele sunt întotdeauna eficiente?

Antibioticele acționează doar asupra bacteriilor și ciupercilor! Bacteriile sunt considerate microorganisme unicelulare. Există câteva mii de specii de bacterii, dintre care unele coexistă destul de normal cu oamenii - mai mult de 20 de specii de bacterii trăiesc în intestinul gros. Unele bacterii sunt oportuniste - provoacă boli numai în anumite condiții, de exemplu, atunci când intră într-un habitat atipic. De exemplu, de foarte multe ori prostatita este cauzată de E. coli, care intră pe calea ascendentă dinspre rect.

Notă: Antibioticele sunt complet ineficiente boli virale. Virușii sunt de multe ori mai mici decât bacteriile, iar antibioticele pur și simplu nu au un punct de aplicare pentru capacitatea lor. De aceea antibioticele nu au nici un efect asupra racelilor, deoarece racelile in 99% din cazuri sunt cauzate de virusi.

Antibioticele pentru tuse și bronșită pot fi eficiente dacă sunt cauzate de bacterii. Numai un medic poate afla ce cauzează boala - pentru aceasta el prescrie teste de sânge și, dacă este necesar, o examinare a sputei dacă aceasta iese.

Important:Este inacceptabil să-ți prescrii antibiotice! Acest lucru va duce doar la dezvoltarea rezistenței unor agenți patogeni și data viitoare boala va fi mult mai greu de vindecat.

Desigur, antibioticele sunt eficiente pentru - această boală este exclusiv de natură bacteriană, cauzată de streptococi sau stafilococi. Pentru a trata durerea în gât, se folosesc cele mai simple antibiotice - penicilină, eritromicină. Cel mai important lucru în tratamentul anginei pectorale este respectarea frecvenței de dozare și a duratei tratamentului - cel puțin 7 zile. Nu trebuie să încetați să luați medicamentul imediat după debutul afecțiunii, care este de obicei observată în a 3-4-a zi. Amigdalita adevărată nu trebuie confundată cu amigdalita, care poate fi de origine virală.

Notă: durerea în gât netratată poate provoca febră reumatismală acută sau!

Pneumonia (pneumonia) poate fi atât de origine bacteriană, cât și virală. Bacteriile provoacă pneumonie în 80% din cazuri, așa că chiar și atunci când sunt prescrise empiric, antibioticele pentru pneumonie au efect bun. Pentru pneumonia virală, antibioticele nu au efect terapeutic, deși împiedică flora bacteriană să se alăture procesului inflamator.

Antibiotice și alcool

Consumul de alcool și antibiotice în același timp într-o perioadă scurtă de timp nu duce la nimic bun. Unele medicamente sunt descompuse în ficat, la fel ca alcoolul. Prezența antibioticelor și a alcoolului în sânge pune o presiune puternică asupra ficatului - pur și simplu nu are timp să neutralizeze alcoolul etilic. Ca urmare, probabilitatea de dezvoltare simptome neplăcute: greață, vărsături, tulburări intestinale.

Important: o serie de medicamente interacționează cu alcoolul la nivel chimic, în urma căruia efectul terapeutic este direct redus. Aceste medicamente includ metronidazolul, cloramfenicolul, cefoperazona și o serie de altele. Utilizarea concomitentă a alcoolului și a acestor medicamente nu numai că poate reduce efectul terapeutic, dar poate duce și la dificultăți de respirație, convulsii și moarte.

Desigur, unele antibiotice pot fi luate în timpul consumului de alcool, dar de ce să vă riscați sănătatea? Este mai bine să vă abțineți de la băuturile alcoolice pentru o perioadă scurtă de timp - cursul terapiei antibacteriene depășește rar 1,5-2 săptămâni.

Antibiotice în timpul sarcinii

Femeile însărcinate suferă de boli infecțioase nu mai rar decât toți ceilalți. Dar tratarea femeilor însărcinate cu antibiotice este foarte dificilă. În corpul unei femei însărcinate, fătul crește și se dezvoltă - copilul nenăscut, foarte sensibil la multe chimicale. Intrarea antibioticelor în organismul în curs de dezvoltare poate provoca dezvoltarea malformațiilor fetale și leziuni toxice ale sistemului nervos central al fătului.

În primul trimestru de sarcină, este recomandabil să evitați cu totul utilizarea antibioticelor. În al doilea și al treilea trimestru, utilizarea lor este mai sigură, dar ar trebui și limitată, dacă este posibil.

O femeie însărcinată nu poate refuza să prescrie antibiotice pentru următoarele boli:

  • Pneumonie;
  • angina pectorală;
  • răni infectate;
  • infectii specifice: bruceloza, borelioza;
  • infectii cu transmitere sexuala: , .

Ce antibiotice pot fi prescrise unei femei însărcinate?

Penicilina, medicamentele cefalosporine, eritromicina și josamicina nu au aproape niciun efect asupra fătului. Penicilina, deși trece prin placentă, nu are un efect negativ asupra fătului. Cefalosporina și alte medicamente numite pătrund în placentă în concentrații extrem de scăzute și nu sunt capabile să dăuneze copilului nenăscut.

Medicamentele sigure condiționat includ metronidazol, gentamicina și azitromicină. Sunt prescrise numai din motive de sănătate, atunci când beneficiul pentru femeie depășește riscul pentru copil. Astfel de situații includ pneumonia severă, sepsisul și alte infecții severe, în care, fără antibiotice, o femeie poate muri pur și simplu.

Ce medicamente nu trebuie prescrise în timpul sarcinii?

Următoarele medicamente nu trebuie utilizate la femeile însărcinate:

  • aminoglicozide– poate duce la surditate congenitală (cu excepția gentamicinei);
  • claritromicină, roxitromicină– în experimente au avut un efect toxic asupra embrionilor de animale;
  • fluorochinolone;
  • tetraciclină– perturbă formarea sistemul ososși dinții;
  • cloramfenicol– periculos pentru mai tarziu sarcina datorita inhibarii functiilor maduvei osoase la copil.

Pentru unele medicamente antibacteriene nu există date privind efectele negative asupra fătului. Acest lucru este explicat simplu - experimentele nu sunt efectuate pe femeile însărcinate pentru a determina toxicitatea medicamentelor. Experimentele pe animale nu ne permit să excludem toate efectele negative cu o certitudine de 100%, deoarece metabolismul medicamentelor la oameni și animale poate diferi semnificativ.

Vă rugăm să rețineți că, de asemenea, ar trebui să încetați să luați antibiotice sau să vă schimbați planurile pentru concepție. Unele medicamente au un efect cumulativ - se pot acumula în corpul unei femei și, pentru o perioadă de timp după terminarea cursului de tratament, sunt metabolizate și eliminate treptat. Se recomandă să rămâneți însărcinată nu mai devreme de 2-3 săptămâni de la terminarea tratamentului cu antibiotice.

Consecințele consumului de antibiotice

Intrarea antibioticelor în corpul uman duce nu numai la distrugerea bacteriilor patogene. La fel ca toate substanțele chimice străine, antibioticele au un efect sistemic - într-o măsură sau alta afectează toate sistemele corpului.

Există mai multe grupuri de efecte secundare ale antibioticelor:

Reactii alergice

Aproape orice antibiotic poate provoca alergii. Severitatea reacției variază: erupție cutanată pe corp, edem Quincke (angioedem), șoc anafilactic. În timp ce o erupție alergică este practic inofensivă, șocul anafilactic poate fi fatal. Riscul de șoc este mult mai mare în cazul injecțiilor cu antibiotice, motiv pentru care injecțiile trebuie făcute numai când institutii medicale– acolo se poate acorda asistență de urgență.

Antibiotice și alte medicamente antimicrobiene care provoacă reacții alergice încrucișate:

Reacții toxice

Antibioticele pot afecta multe organe, dar ficatul este cel mai susceptibil la efectele lor - hepatita toxică poate apărea în timpul terapiei cu antibiotice. Anumite medicamente au un efect toxic selectiv asupra altor organe: aminoglicozide - asupra aparatului auditiv (provoacă surditate); tetraciclinele inhibă creșterea oaselor la copii.

Notă: Toxicitatea unui medicament depinde de obicei de doza acestuia, dar în caz de intoleranță individuală, uneori, doze mai mici sunt suficiente pentru a produce un efect.

Efecte asupra tractului gastrointestinal

Când iau anumite antibiotice, pacienții se plâng adesea de dureri de stomac, greață, vărsături și tulburări ale scaunului (diaree). Aceste reacții sunt cel mai adesea cauzate de efectul iritant local al medicamentelor. Efectul specific al antibioticelor asupra florei intestinale duce la tulburări funcționale ale activității sale, care este cel mai adesea însoțită de diaree. Această afecțiune se numește diaree asociată cu antibiotice, care este cunoscută în mod popular sub numele de disbioză după antibiotice.

Alte efecte secundare

Alte reacții adverse includ:

  • imunosupresie;
  • apariția unor tulpini de microorganisme rezistente la antibiotice;
  • suprainfecție – o afecțiune în care microbii rezistenți la un anumit antibiotic sunt activați, ducând la apariția unei noi boli;
  • încălcarea metabolismului vitaminelor - cauzată de inhibarea florei naturale a colonului, care sintetizează unele vitamine B;
  • Bacteroliza Jarisch-Herxheimer este o reacție care apare la utilizarea medicamentelor bactericide, când, ca urmare a morții simultane a unui număr mare de bacterii, un număr mare de toxine sunt eliberate în sânge. Reacția este similară clinic cu șocul.

Pot fi utilizate antibioticele profilactic?

Autoeducarea în domeniul tratamentului a dus la faptul că multe paciente, în special tinerele mame, încearcă să-și prescrie singure (sau copilul lor) un antibiotic la cel mai mic semn de răceală. Antibioticele nu au actiune preventiva– tratează cauza bolii, adică elimină microorganismele, iar dacă lipsesc, apar doar efectele secundare ale medicamentelor.

Există un număr limitat de situații în care antibioticele sunt administrate înainte de manifestările clinice ale infecției, pentru a preveni aceasta:

  • interventie chirurgicala– în acest caz, antibioticul prezent în sânge și țesuturi previne dezvoltarea infecției. De regulă, este suficientă o singură doză de medicament administrată cu 30-40 de minute înainte de intervenție. Uneori, chiar și după o apendicectomie, antibioticele nu sunt injectate în perioada postoperatorie. Dupa "curatare" operatii chirurgicale Antibioticele nu sunt prescrise deloc.
  • răni sau răni majore (fracturi deschise, contaminarea solului rănii). În acest caz, este absolut evident că o infecție a intrat în rană și ar trebui să fie „zdrobită” înainte de a se manifesta;
  • prevenirea de urgență a sifilisului efectuate în timpul contactului sexual neprotejat cu o persoană potențial bolnavă, precum și în rândul lucrătorilor sanitari care au intrat în contact cu mucoasa sângele unei persoane infectate sau alt fluid biologic;
  • penicilina poate fi prescrisă copiilor pentru prevenirea febrei reumatice, care este o complicație a amigdalitei.

Antibiotice pentru copii

Utilizarea antibioticelor la copii nu este în general diferită de utilizarea lor la alte grupuri de oameni. Pentru copiii mici, pediatrii prescriu cel mai adesea antibiotice în sirop. Acest forma de dozare mai comod de luat, spre deosebire de injecții, complet nedureroase. Copiilor mai mari li se pot prescrie antibiotice sub formă de tablete și capsule. În cazurile severe de infecție, aceștia trec pe calea de administrare parenterală - injecții.

Important: Principala caracteristică a utilizării antibioticelor în pediatrie este doza - copiilor li se prescriu doze mai mici, deoarece medicamentul este calculat în funcție de kilogramul de greutate corporală.

Antibioticele sunt medicamente foarte eficiente, dar în același timp au un număr mare de efecte secundare. Pentru a fi vindecați cu ajutorul lor și pentru a nu vă face rău organismului, ele trebuie luate numai conform prescripției medicului.

Ce tipuri de antibiotice există? În ce cazuri este necesară administrarea de antibiotice și în ce cazuri este periculos? Principalele reguli ale tratamentului cu antibiotice sunt explicate de medicul pediatru Dr. Komarovsky:

Gudkov Roman, resuscitator